You are on page 1of 94

ARNAB SARKAR

ARNAB SARKAR

SQA Stability July 2005


1. A vessel is to transit a canal with a minimum clearance of 0.30m under a bridge, the
underside of which is 20.24m above the waterline.
Present draughts in FW (RD 1.000): Forward 5.22m Aft 6.38m
The foremast is 112m FOAP and extends 25.92m above the keel.
The aft mast is 37m FOAP and extends 26.94m above the keel.
(Assume masts are perpendicular to the waterline throughout)
Using the Stability Data Booklet, calculate EACH of the following:
(a) The final draughts forward and aft in order to pass under the bridge with
minimum clearance;
(b) The minimum weight of ballast to load in order to pass under the bridge with
minimum clearance.
a) Air draught required = 20.24 0.30 = 19.94m
=
=
=

Foremast
25.920m
19.940m
5.980m

Trim at Masts

7.000 5.980 = 1.020m by stern

Correction for Draught F


Trim at Masts

LBP Dist. of Foremast from AP


Dist. of Foremast from AP Dist. of Aft mast from AP

Correction for Draught F

(137.5 112.0) x 1.020


(112.0 37.0)

Correction for Draught A


Trim at Masts

Dist. of Aft mast from AP


Dist. of Foremast from AP Dist. of Aft mast from AP

Correction for Draught A

37.0
x 1.020 = 0.503m
(112.0 37.0)

Required draught
Required correction
Draft at perpendiculars

=
=
=

Height above the keel


Air draught required
Draught at Masts

Aft Mast
26.940m
19.940m
7.000m

Correction for Draught F

= 0.347m

Correction for Draught A

Calculate required draft at perpendiculars


Foremast
5.980m
(-) 0.347m
5.633m

Aft Mast
7.000m
(+) 0.503m
7.503m

b) Calculate Initial Displacement


Initial trim
Initial AMD

= 6.380 5.220
= 1.160m by stern
= (6.380 + 5.220)
= 5.800m
2
With AMD, from hydrostatic tables,
LCF = 68.57m
TMD = 6.380 (1.160 x 68.57)
= 5.802m
137.5
Displacement TMDi

= 11559 + (219 x 0.002)


0.10

= 11563.38 t 11563 t

ARNAB SARKAR

SQA Stability July 2005


Calculate Final Displacement
Final trim
Final AMD

= 7.503 5.633
= (7.503 + 5.633)
2

= 1.870m by stern
= 6.568m

With AMD, from hydrostatic tables,


LCF = 67.90 (0.11 x 0.068)
= 67.83m
0.10
TMD = 7.503 (1.870 x 67.83)
= 6.581m
137.5
Displacement TMDf = 13102 + (222 x 0.081)
0.10

= 13281.82 t 13282 t

Calculate minimum weight of ballast to load in order to achieve required trim


Ballast to load

= Displacement TMDf Displacement TMDi


= 13282 11563 = 1719 t

2. A vessel is planning to enter dry-dock in dock water of relative density 1.015.


Present draughts: Forward 3.00m Aft 3.80m KG 10.50m
a) Using the Stability Data Booklet, calculate the maximum trim at which the vessel
can enter dry-dock so as to maintain a GM of at least 0.10m at the critical instant.
Assume KM remains constant.
b) Explain why it is beneficial to have a small stern trim when entering dry-dock.
a) Calculate Initial TMD
Initial Trim
Initial AMD

= 3.800 3.000
= 0.800m
= (3.800 + 3.000)
= 3.400m
2
With AMD, from hydrostatic tables,
LCF = 70.20m
TMD = 3.800 (0.800 x 70.20)
= 3.392m
137.5
Calculate Initial , MCTC, LCF & GM
With TMD, from hydrostatic tables,
FW

= 6261 + (205 x 0.092)


0.10
= 6449.6 x 1.015

= 6449.6 t

MCTC = 139.4 + (1 x 0.092)


0.10
MCTC DW = 140.3 x 1.015

= 140.3tm

= 70.27 (0.07 x 0.092)


0.10

= 70.21m

KM T = 11.18 (0.23 x 0.092)


0.10
KG
= 10.50m
GM = 0.47m

= 10.97m

DW

LCF

= 6546.3 t

= 142.4tm

ARNAB SARKAR

SQA Stability July 2005


Calculate required Loss of GM
Initial GM
= 0.47m
Required GM
= 0.10m
Reqd. Loss of GM = 0.37m

Calculate the required P Force with the required Loss of GM


Loss of GM
P

= P x KM T

= 0.37 x 6546.3
10.97

= 220.8 t

Calculate the required trim with the P Force


P

= trim x MCTC
LCF
Reqd. Trim = 220.8 x 70.21
142.4

= 108.86cms = 1.089 m by stern

b) It is beneficial to have a small stern trim when entering dry-dock because this allows:
1)
2)
3)
4)

A more gradual transfer of weight and thus a more gradual reduction in stability.
The vessels stern to be used as a pivot to align the keel along the blocks.
Some of the shores to be positioned early, working from aft to forward.
The stern frame is specially strengthened to accept the forces exerted on it during drydocking but there will be a maximum limit that must not be exceeded. If the P force
becomes too great, structural damage will occur.
5) Greater the trim, greater the P force and greater loss of GM.
Therefore a slight stern trim is beneficial.
3. A vessels loaded particulars in salt water are as follows:
Displacement 16000 t
Fluid KG 8.20m
Using the Stability Data Booklet, compare the vessels stability with ALL the minimum
stability criteria required by the current Load Line Regulations, commenting on the
result.
= 16000 t

KG F = 8.20m

Determine Angle of Flooding ( F ) for = 16000 t


F

= 44.5

Derive the KN values from the Hydrostatic Particulars

Angle of Heel ()

(Tonnes)

12

20

30

40

50

60

75

16000

1.73

2.98

4.40

5.60

6.35

6.83

7.00

Calculate the GZ values


Heel ()

KN (m)

KG Sin (m)

GZ (m)

12
20
30
40

1.73
2.98
4.40
5.60

1.70
2.80
4.10
5.27

0.03
0.18
0.30
0.33

ARNAB SARKAR

SQA Stability July 2005


50
6.35
6.28
60
6.83
7.10
75
7.00
7.92
Calculate the vessels GM F
From hydrostatic particulars, using = 16000 t in SW

0.07
- 0.27
- 0.92

KM T = 8.33 + (0.01 x 24) = 8.33m


236
GM = 8.33 8.20
= 0.13m
Construct the GZ Curve

GZ Curve
0.6
0.4
0.2
0
0

10

20

30

40

50

60

-0.2
-0.4
-0.6
-0.8
-1

Calculate the area under the GZ Curve from 0 30


Angle ()
0
10
20
30

GZ (m)
0
0.02
0.18
0.30

SM
1
3
3
1

Product
0
0.06
0.54
0.30
0.90

Area 0 30

= 3 x 10 x 0.90
= 0.059 m-rad
8
57.3
Calculate the area under the GZ Curve from 0 40
Angle ()
0
10
20
30
40

GZ (m)
0
0.02
0.18
0.30
0.33

SM
1
4
2
4
1

Product
0
0.08
0.36
1.20
0.33
1.97

70

80

ARNAB SARKAR

SQA Stability July 2005


Area 0 40

= 1 x 10 x 1.97
= 0.115 m-rad
3
57.3
Calculate the area under the GZ Curve from 30 40
Area 30 40

= 0.115 0.059 = 0.056 m-rad

Summary

Actual

L.L. Regulation

Complies or Not

Area 0 30

0.059 m-r

Not less than 0.055 m-r

Yes

Area 0 40

0.115 m-r

Not less than 0.09 m-r

Yes

Area 30 40

0.056 m-r

Not less than 0.03 m-r

Yes

0.33m

At least 0.20m

Yes

40

Equal to or greater than 30

Yes

0.13m

Not less than 0.15m

No

Max. GZ
Max. GZ Angle
GM F

ARNAB SARKAR

SQA Stability November 2005


1. A vessels present particulars are as follows:
Forward draught: 8.00m Aft draught: 9.00m in salt water
A total of 320 t of cargo, LCG 70.00m FOAP, is to be discharged immediately and then
the vessel is to proceed to an upriver berth where the relative density of the dock water
is 1.005.
During the passage the following items of deadweight are consumed:
90 t of Heavy Fuel Oil
18 t of Diesel Oil
18 t of Diesel Oil
12 t of Fresh Water

from No.3 DB Centre tank


from No.4 DB Port tank
from No.4 DB Starboard tank
from After Fresh Water tank

Using the Stability Booklet Data, calculate the draughts fore and aft, on arrival at the
upriver berth.
1. Forward Draught

SW

= 8.000m

Aft Draught

SW

= 9.000m

Calculate trim & AMD


Trim = 9.000 8.000
AMD = (9.000 + 8.000)
2

= 1.000m by stern
= 8.500m

Calculate LCF & TMD


LCF = 65.95m FOAP
TMD = 9.000 (65.95 x 1.000)
137.5

= 8.520m

From hydrostatic tables, using TMD = 8.520m, calculate Initial , MCTC SW & LCB

= 18119 + (240 x 0.02)


0.10

= 18167 t

MCTC SW

= 206.4 + (1.3 x 0.02)


0.10

= 206.7 tm

LCB

= 69.36 (0.05 x 0.02)


0.10

= 69.35m FOAP

Calculate Initial LCG


1.000 x 100
LCG

= 18167 x (69.35 LCG)


206.7
= 68.21m FOAP

Calculate Longitudinal Moments about the LCG


Condition
Initial
Cargo discharged
HFO 3DB (C)
DO 4DB (P)
DO 4DB (S)
FW AFWT
Final

Weight (t)
18167
(-) 320
(-) 90
(-) 18
(-) 18
(-) 12
17709

Final LCG = 68.33m FOAP

LCG (m)
68.21
70.00
57.02
35.66
35.50
28.67
68.33

Longitudinal Moments (tm)


1239171.07
(-)
22400.00
(-)
5131.80
(-)
641.88
(-)
639.00
(-)
344.04
1210014.35

ARNAB SARKAR

SQA Stability November 2005


Final SW
Volume in DW
FW

= 17709 t
= 17709
1.005
= 17621 t

= 17620.9 m3 17621 m3

From hydrostatic tables, using FW = 17621 t, calculate TMD, MCTCDW , LCB & LCF
TMD
MCTC FW
MCTC DW

= 8.40 + (179 x 0.10)


235

= 8.476m

= 200.1 + (179 x 1.3)


235
= 201.1 x 1.005 = 202.1 tm

= 201.1 tm

LCB

= 69.40 (179 x 0.04)


235

= 69.37m FOAP

LCF

= 66.02 (179 x 0.07)


235

= 65.97m FOAP

Calculate change of trim


trim

= 17709 x (69.37 68.33)


202.1

= 91.130 cms = 0.911 m by stern

COT A

= 0.911 x 65.97
137.5

= 0.437 m

COT F

= 0.911 0.437

= 0.474 m

Calculate Final Draughts


Draught
Draught

Fwd
Aft

= 8.476 0.474 = 8.002 m


= 8.476 + 0.437 = 8.913 m

2. A vessel is to load a cargo of grain (Stowage Factor 1.57m3/t).


Initial displacement 6400 t. Initial KG 6.48m.
The tween decks are to be loaded as follows:
No.1 TD
No.2 TD
No.3 TD
No.4 TD

Full
Part Full Ullage 1.25m
Part Full Ullage 3.00m
Full

The Stability Data Booklet provides the necessary cargo compartment data for the
vessel.
a) With the aid of Maximum Permissible Grain Heeling Moment Table included in the
Stability Data Booklet, determine whether the vessel complies with the minimum
criteria specified in the International Grain Code (IMO).
b) Calculate the ships approximate angle of list in the event of the grain shifting as
assumed by the International Grain Code (IMO).
2. a)

Initial
Initial KG
SF

= 6400 t
= 6.48m
= 1.57 m3/t

Calculate the Final , KG & VHM

ARNAB SARKAR

SQA Stability November 2005


Comp.

Ullage
(m)

Volume
(m3)

SF
(m3/t)

Initial

Vert. Mom.
(tm)
41472

VHM
(tm)

6400

KG
(m)
6.48

Tonnes

No.1 TD

Full

1695

1.57

1080

11.26

12161

352

No.2 TD

1.25

1560

1.57

994

10.65

10586

1642

No.3 TD

3.00

758

1.57

483

10.13

4893

3339

No.4 TD

Full

1674

1.57

1066

10.57

11268

604

No.1 Hold

Full

2215

1.57

1411

5.09

7182

410

No.2 Hold

Full

4672

1.57

2976

4.95

14731

1285

No.3 Hold

Full

1742

1.57

1110

4.94

5483

475

No.4 Hold

Full

3474

1.57

2213

4.95

10954

910

No.5 Hold

Full

2605

1.57

1659

8.76

14533

455

19392

6.87

133263

9472

Final
Calculate AHM
AHM = 9472
1.57

= 6033 tm

Calculate Maximum Permissible Heeling Moment


= 19392 t

KG F = 6.87m

Interpolation

KG F = 6.80

KG F = 6.87

KG F = 6.90

19000

7577

7283.0

7157

19392
19500

7428.3
7770

7468.3

Holding = 19000 t constant, using variable KG F = 6.87m,


7577 (420 x 0.07) = 7283.0 tm
0.10
Holding = 19500 t constant, using variable KG F = 6.87m,
7770 (431 x 0.07) = 7468.3 tm
0.10
Holding KG F = 6.87m constant, using variable = 19392 t,
7283 + (185.3 x 392) = 7428.3 tm
500
PHM for = 19392 t & KG F = 6.87m = 7428.3 tm

As AHM < PHM, vessel complies with the minimum criteria.


b) Calculate approximate angle of list in event of grain shift
Angle of Heel = 6033 x 12 = 9.7
7428.3

7339

ARNAB SARKAR

SQA Stability November 2005


3. a) A vessel, initially upright, is to carry out an inclining test.
Present displacement 5700 t, KM 10.83m.
Total weights on board during the experiment:
Ballast 370 t, KG 3.47m, tank full.
Bunkers 165 t, KG 3.98m, free surface moment 956 tm.
Water 95 t, KG 4.44m, slack tank. Free surface moment 910 tm.
Boiler water 19 t, KG 4.18m, free surface moment 102 tm.
Inclining weights 56 t, KG 8.44m
A deck crane weights 19 t and still ashore will be fitted on the vessel at a KG of 9.74m
at a later date.
The plumblines have an effective vertical length of 7.85m. The inclining weights are
shifted transversely 7.0m on each occasion and the mean horizontal deflection of the
plumb line is 0.65m.
Calculate the vessels lightship KG.
b) Explain why a vessels lightship KG may change over a period of time.
3. a)

= 5700 t
Plumb length = 7.85m
Trans shift
= 7.00m

KM
Deflection

= 10.83m
= 0.65m

Calculate GM F
GM F = w x d x plum length
W x deflection

= 56 x 7 x 7.85
5700 x 0.65

= 0.83m

Calculate KG F
KM = 10.83m
GM F = 0.83 m
KG F = 10.00m

Calculating moments about the C L ,


Condition
Initial
(-) Ballast
(-) Bunkers
(-) FSM Bunkers
(-) Water
(-) FSM Water
(-) Inclining Weight
(+) Deck Crane

Weight (t)
5700
(-) 370
(-) 165

KG (m)
10.00
3.47
3.98

(-) 95

4.44

(-) 56
19
5033

8.44
9.74

Lightship KG

= 52484.02
5033

= 10.43m

Vertical Moments (tm)


57000
(-)
1283.9
(-)
656.7
(-)
956.0
(-)
421.8
(-)
910.0
(-)
472.64
(+)
185.06
52484.02

b) A vessels lightship KG may change over a period of time due to the following reasons:
Constant of the vessel keep changing due to accumulated sludge in fuel tanks, mud
and rust in ballast tanks (un-pumpables)
Various stores remaining unconsumed might add to the constant.
Any structural changes will affect the light ship KG and light ship displacement

ARNAB SARKAR

SQA Stability November 2005


Lightship KG for a passenger vessel will change considerably over a period of time
mainly because of the left over baggage etc. will accumulate over a period of time
and add to the constant considerably.

ARNAB SARKAR

SQA Stability March 2006


1. A box shaped vessel floating on even keel in salt water has the following particulars:
Length:
170.0m
Breadth:
22.0m
Draught:
9.2m
KG:
6.5m
There is an empty forward end compartment, length 21.0m extending the full width of
the vessel.
Calculate the draughts forward and aft, if this compartment is bilged.
Volume of the vessel before bilging = 170 x 22 x 9.2
Displacement of the vessel
= 34408 x 1.025

= 34408 m3
= 35268.2 t

Volume of bilged compartment


Intact WPA

= 21 x 22 x 9.2
= (170 21) x 22

= 4250.4 m3
= 3278 m2

Sinkage

= 4250.4
3278

= 1.297m

= 9.200 + 1.297

= 10.497m

Calculate Bilged TMD

Bilged TMD
Calculate BB H
BB H

= 21
2

= 10.5m aft

Trimming Moment

= 10.5 x 35268.2

KB

= 5.25m

= 370316.1 tm

Calculate GM L

= 10.497
2

BM L = (170 21)3 x 22
12 x 34408

= 176.25m

KM L = 176.25 + 5.25
GM L = 181.50 6.50

= 181.50m
= 175.00m

MCTC = 35268.2 x 175.00


100 x 170

= 363.1 tm

Calculate MCTC

Calculate COTF & COT A


COT = 370316.1
363.1
COT A

COT F

= 1019.9cms or 10.199m by forward

= 10.199 x 74.5
170
= 10.199 4.470

= 4.470m
= 5.729m

Calculate Final Draughts


Bilged TMD
COT F/A
Final Draughts

(+)

Forward
10.497m
5.729m
16.226m

(-)

Aft
10.497m
4.470m
6.027m

ARNAB SARKAR

SQA Stability March 2006


2. A vessel initially upright and on an even keel, has the following particulars:
Draught (in salt water): 6.80m
Breadth: 20.42m
KG: 7.88m
Further particulars of the vessel can be found in the Stability Data Booklet.
The vessels heavy lift derrick is to be used to discharge a 60 tonne tank from a
centreline position, KG 5.23m. The derrick head is 29.28m above the keel and 15.80m
out of the centreline when plumbing over side.
a) Calculate the maximum list angle.
b) Calculate the increase in draught when the vessel is at maximum list angle as
calculated in Q 2(a), assuming rectangular cross section midships.
c) Calculate the maximum allowable KG prior to discharging the tank in order to
limit the list angle to 5.
a) The maximum angle of list occurs when the tank will be just above quay hanging on the
derrick. The KG of the tank will act on the derrick head.
6.80m in SW

= 14115 t

&

KM 6.80m in SW = 8.36m

Calculate GGH
GG H

= 60 x 15.80 = 0.067m
14115

Calculate moments about the keel


Weight
14115
60
60
14115

Distance from keel


7.88
5.23
29.28

Initial
Tank
Tank
Final

(-)
(+)

Final KG

= 112669.2
14115

= 7.982m

= 8.36 7.982

= 0.378m

Tan

= 0.067
0.378
= 10.1

GM

List

List

a) Draught when heeled = (6.800 x Cos 10.1) + (20.42 x Sin 10.1)


2
= 8.485m
Initial Draught
= 6.800m
Increase in draught = 8.485 6.800 = 1.685m
b) Limitation of List
Tan 5

Required GM
Required KG

= 5

= 0.067
GM
= 0.766m
= 8.36 0.766 = 7.594m

Assuming maximum allowable KG to be x metres,

(-)
(+)

Moments
111226.2
313.8
1756.8
112669.2

ARNAB SARKAR

SQA Stability March 2006


Calculate moments about the keel
Initial
Tank
Tank
Final

(-)
(+)

Weight
14115
60
60
14115

Required KG prior discharging

Distance from keel


x
5.23
29.28
= (14115x + 1443)
14115

(-)
(+)

Moments
14115x
313.8
1756.8
(14115x + 1443)

= 7.594

Hence, required KG = 7.492m

3. A vessel is floating in dock water of RD 1.012.


Present draughts are as follows:
Forward (mean) 6.500m; Midship (port) 7.160m; Midship (stbd) 7.140m
Aft (mean) 7.700m;
The draught marks are displaced as follows:
Forward
: 2.40m aft of the FP
Aft
: 3.60m forward of AP
Midship
: 1.96m aft of amidships line
Using the Stability Data Booklet and Worksheet Q3, determine the vessels
displacement.
Draught forward
Trim
FP Correction
Draught at FP
Draught aft
AP Correction
Draught at AP
True Trim

= 6.500m
= 7.700 6.500
= 2.40 x 1.200
137.5
= 6.500 0.021
= 7.700m
= 3.60 x 1.200
137.5
= 7.700 + 0.031
= 7.731 6.479

Draught (M) Port


Draught (M) Stbd

= 7.160m
= 7.140m

Draught (M) Mean

= 7.150m

= 1.200m by stern
= 0.021m
= 6.479m
= 0.031m
= 7.731m
= 1.252m by stern

Amidships Line Corr. = 1.96 x 1.252


= 0.018m
137.5
Draught at Midships = 7.150 0.018
= 7.132m
Corr. (M) draught = 6.479 + (6 x 7.132) + 7.731
8

= 7.125m

From hydrostatic particulars using draught 7.125m


TPC
LCF
Displacement

= 23.19 + (0.07 x 0.025)


0.10
= 67.24 (0.11 x 0.025)
0.10
= 14808 + (232 x 0.025)
0.10

= 23.21tm
= 67.21m foap
= 14866 t

ARNAB SARKAR

SQA Stability March 2006


Distance of LCF from Midships
1st Trim Correction
MCTC 6.632m
MCTC 7.632m
2nd Trim Correction
Corrected
Dock Water

= 68.75 67.21
= 1.54 x 125.2 x 23.21
137.5

= 1.54m
= 32.546 t

= 178.3 + (1.6 x 0.032)


= 178.8 tm
0.10
= 193.9 + (1.5 x 0.032)
= 194.4 tm
0.10
= 50 x (1.252)2 x 15.6
= 8.892 t
137.5
= 14866 + 32.546 + 8.892
= 14907.438 x 1.012
1.025

= 14907.438 t
= 14718.368 t

ARNAB SARKAR

SQA Stability July 2006


1. A box shaped vessel floating upright on an even keel in salt water has the following
particulars:
Length BP : 150.00m Breadth: 28.00m

Even keel draught: 8.60m KG: 9.20m

The vessel has two longitudinal bulkheads each 9.00m from the side of the vessel.
Calculate the angle of heel if an amidship side compartment 24.00m is bilged.
1. Calculate Sinkage and Bilged Draught
Volume of Buoyancy lost
or, (24 x 9 x 8.600)
or, Sinkage

=
=
=

Bilged draught

Volume of Buoyancy gained


{(150 x 28) (24 x 9)} x Sinkage
1857.6
= 0.466m
3984
8.600 + 0.466 = 9.066m

Calculate new location of LCF from the side XX


Calculate moments of area about the axis XX
Total Area
Bilged Area

Area (m2)
150 x 28 = 4200
24 x 9 = 216
3984

New location of LCF


Distance BB H

Distance from axis XX (m)


14.0
4.5
()

Moments (m4)
58800
972
57828

= 14.515m
= 14.515 14.0 = 0.515m

Calculate Moment of Inertia (I) about the new LCF


= I XX Ad2
3
3
2
LL = [(150 x 28 ) (24 x 9 )] [{(150 x 28) (24 x 9)} x 14.515 ]
3
3
or, I LL = (1097600 5832) 839369.936
or, I LL = 252398.064 m4
I LL
or, I

Calculate bilged BM, KB, KM and GM


BM

252398.064
(150 x 28 x 8.600)
KB
= 9.066
2
KM = 4.533 + 0.593
GM = 11.521 9.200

= 6.988m
= 4.533m
= 11.521m
= 2.321m

Calculate List

Tan = 0.515
2.321
List = 12.5

2. Worksheet Q2 Trim and Stability provides data relevant to a particular condition of


loading in a vessel in salt water.
The Stability Data Booklet provides the necessary hydrostatic data for the vessel.
By completion of the Worksheet Q2 and showing any additional calculations in the
answer book, calculate each of the following:
(a) The effective metacentric height;
(b) The draughts forward and aft.

ARNAB SARKAR

SQA Stability July 2006

CONDITION: BALLAST

Comp.

Grain
Capacity (m3)

Stowage
Factor
(m3/t)

Weight
(t)

KG
(m)

Vertical
Moment
(tm)

All holds

15875

1.94

8183

4.86

1 TD

1265

2.37

534

2 TD

1332

2.53

3 TD

1406

4 TD

1230

Free
Surface
Moment
(tm)

LCG
foap
(m)

Long.
Moment (tm)

39769

68.1

557262

10.98

5863

114.0

60876

526

10.42

5481

95.6

50286

2.49

565

10.33

5836

74.0

41810

2.66

462

10.18

4703

55.0

25410

Oil Fuel

856

1786

868

31020

FW

84

618

75

4550

FSM
Lightship

3831

8.21

31453

61.66

236219

15041

6.41

96452

66.98

1007433

HYDROSTATICS

True Mean Draught =


7.20 + (0.10 x 1) = 7.200m
234

LCB foap =
69.94m

LCF foap =
67.13m

LBP = 137.5m

MCTC = 187.8 tm

KM T = 8.33m

GM = 1.92m

LCB from LCG = 69.94 66.98 = 2.96m (As LCB > LCG, therefore vessel is trimmed by the stern)
Trim between Perpendiculars: 15041 x 2.96 = 237.1 cm or 2.371m
187.8
COT A = 2.371 x 67.13 = 1.158m
137.5
COT F = 2.371 1.158 = 1.213m
Draughts:
Forward: 7.20 1.213 = 5.987m

Aft = 7.20 + 1.158 = 8.358m

3. A vessel has the following particulars:


Displacement 17000 t, Even keel draught 8.03m, Maximum breadth 21.00m
KG 8.00m
KM 8.35m KB 4.17m
(a) Explain why this vessel will heel on turning.
(b) Calculate the angle and direction of heel when turning to starboard in a circle of
diameter 500m at a speed of 17.5 knots.
Note: assume 1 nautical mile = 1852m, and g = 9.81 m/sec2
(c) Calculate the new maximum draught during the turn in Q3 (b), assuming the
amidships cross-section can be considered rectangular.
(a) To turn a ship the rudder is turned and this causes a force to act on the ship towards the
centre of the turn. This is the centripetal force and it acts at the ships centre of lateral
resistance which is assumed to be at the centroid of the underwater volume (B).
The inertia of the vessel resists this change of direction with an equal and opposite force
which acts at the ships centre of gravity (G) termed as the centrifugal force.

ARNAB SARKAR

SQA Stability July 2006

The vessel will be in equilibrium at its angle of heel when:


The righting moment = The heeling moment
(b) BG

= KG KB
= 8.00 4.17 = 3.83m

Speed during turning (v)


Radius of turn

GM

= KM KG
= 8.35 8.00 = 0.35m

= 17.5 Knots = 17.5 x 1852 = 9.00 m/sec


3600
= 500 = 250m
2

Angle of heel during turning = tan = (9.00)2 x 3.83


9.81 x 250 x 0.35
or, = 19.9 to port
(c) Draught when heeled = (8.03 x Cos 19.9) + (0.5 x 21.00 x Sin 19.9)
= 11.124m

ARNAB SARKAR

SQA Stability November 2006


1. A vessel is floating in salt water at draughts 7.80m forward and 8.24m aft. The vessel is
to load cargo so as to finish on an even keel at a load draught of 8.68m. Two spaces
available: No. 1 hold, LCG 118m foap; No. 5 hold, LCG 15m foap.
Using the Stability Data Booklet, calculate EACH of the following:
(a) The total weight of cargo to load;
(b) The weight of cargo to load in each compartment.
(a) Calculate AMD and Trim
AMD = 7.800 + 8.240
2

= 8.020m

&

Trim

= 8.240 7.800

Calculate LCF (from hydrostatic particulars) and TMD


LCF

= 66.33 (0.08 x 0.02)


0.10
TMD = 8.240 (0.440 x 66.31)
137.5

= 66.31m
= 8.028m

Calculate initial , MCTC and LCB


Initial
MCTC
LCB

= 16922 + (239 x 0.028)


0.10
= 199.6 + (1.4 x 0.028)
0.10
= 69.58 (0.04 x 0.028)
0.10

= 16989 t
= 200.0 tm
= 69.57m foap

Calculate initial LCG


0.440 x 100
LCG

= 16989 x (69.57 LCG)


200.0
= 69.05m foap

Calculate final , MCTC, & LCB using Even Keel draught = 8.680m
Final
MCTC
LCB

= 18359 + (242 x 0.08)


0.10
= 207.7 + (0.3 x 0.08)
0.10
= 69.31 (0.04 x 0.08)
0.10

= 18553 t
= 207.9 tm
= 69.28m foap

Total cargo loaded = 18553 16989 = 1564 t

Assume that w tonnes of cargo is loaded in No.1 Hold,


Cargo loaded in No.5 Hold = (1564 w) tonnes.
Calculating longitudinal moments about the AP
Weight (t)
16989
w
(1564 w)
18553

LCG (m)
69.05
118.0
15.0

Longitudinal Moments (tm)


1173090.45
118w
(23460 15w)
(1196550.45 103w)

As the vessel is finally in even keel therefore, LCB = LCG = 69.28m foap

= 0.440m

ARNAB SARKAR

SQA Stability November 2006


69.28
w

= (1196550.45 103w)
18553
= 862 t

Cargo loaded in No.1 Hold = 862 t


Cargo loaded in No.5 Hold = 1564 862 = 702 t

2. A vessel is floating at an even keel draught of 7.25m in salt water. KG 7.20m. An


amidship rectangular deck 28.00m long and extending the full breadth of the vessel is
flooded with salt water to a depth of 0.50m.
Height of deck above keel 9.60m
Using the Stability Data Booklet, calculate EACH of the following:
(a) The fluid GM
(b) The angle of loll
(a) Calculate initial (from hydrostatic particulars) using the Even Keel draught of 7.25m
Initial

= 15040 + (234 x 0.05) = 15157 t


0.10

Calculate weight of the salt water in the flooded compartment


Volume
Weight

= 28.0 x 20.42 x 0.50 = 285.88 m3


= 285.88 x 1.025 = 293 t

FSM caused due to water ingress = 28.0 x (20.42)3 x 1.025 x 15157 = 20364.2 tm
12 x 15157
Calculate the final KG after flooding by taking moments about the Keel
Weights (t)
15157
293
FSM
15450

KG (m)
7.20
9.85

Transverse Moments (tm)


109130.4
2886.1
20364.2
132380.7

Final KG fluid after getting flooded = 8.568m

Calculate final KB & KM using final = 15450 t


Final KB
Final KM
Fluid GM

= 3.80 + (0.05 x 176) = 3.84m


233
= 8.33m
= 8.33 8.568 = (-) 0.238m

(b) Calculate BM T
BM T

= 8.33 3.84 = 4.49m

Calculate Angle of Loll after flooding


Tan = 2 x 0.238
4.49
Angle of loll = 18.0

ARNAB SARKAR

SQA Stability November 2006


3. A vessel is floating in dock water of R.D. 1.016.
Present draughts:
Forward 7.50m;
Midship (P) 8.44m; Midship (S) 8.38m; Aft 8.90m.
The draught marks are displaced as follows:
Forward:
1.86m aft of the FP.
Aft:
2.50m aft of the AP.
(The amidships draught marks are not displaced)
The Stability Data Booklet provides the necessary hydrostatic data for the vessel. By
completion of the Worksheet Q3 and showing any additional calculations workbook,
determine EACH of the following:
(a) The vessels displacement;
(b) The maximum cargo to load for a Summer Load Line Zone.
Note: assume no hog or sag in the Summer Load condition.
(a) Draught forward
Trim
FP Correction
Draught at FP
Draught aft
AP Correction
Draught at AP
True Trim

= 7.500m
= 8.900 7.500
= 1.86 x 1.400
137.5
= 7.500 0.019
= 8.900m
= 2.50 x 1.400
137.5
= 8.900 0.025
= 8.875 7.481

Draught (M) Port


Draught (M) Stbd

= 8.440m
= 8.380m

Draught (M) Mean

= 8.410m

= 1.400m by stern
= 0.019m
= 7.481m
= 0.025m
= 8.875m
= 1.394m by stern

Amidships Line Corr. = Nil


Draught at Midships = 8.410m
Corr. (M) draught = 7.481 + (6 x 8.410) + 8.875
8

= 8.352m

TPC

= 23.99tm

From hydrostatic particulars using draught 8.352m

= 23.96 + (0.06 x 0.052)


0.10
LCF
= 66.10 (0.08 x 0.052)
0.10
Displacement
= 17639 + (239 x 0.052)
0.10
Distance of LCF from Midships
= 68.75 66.06
1st Trim Correction
MCTC 7.910m
MCTC 8.910m
2nd Trim Correction

= 66.06m foap
= 17763.28 t
= 2.69m

= 2.69 x 140.0 x 23.99


137.5
= 198.2 + (1.4 x 0.010)
0.10
= 211.5 + (1.2 x 0.010)
0.10
2
= 50 x (1.394) x 13.28
137.5

= 65.706 t

= 198.34 tm
= 211.62 tm
= 9.384 t

ARNAB SARKAR

SQA Stability November 2006


Corrected
Dock Water

= 17763.28 + 65.706 + 9.384


= 17838.37 x 1.016
1.025

(b) Summer load displacement


Maximum cargo to load

= 19006 t
= 19006 17681.7 = 1324.3 t

= 17838.37 t
= 17681.7 t

ARNAB SARKAR

SQA Stability March 2007


1. A vessel is floating upright and is to load TWO weights using the ships own derrick.
The maximum allowable list is 4 degrees.
Initial draughts: 7.30m, forward and aft, in fresh water.
Derrick head 26.5m above the keel.
Two weights, each 42 tonne, are on the quay 17.5m from the vessels centreline.
Stowage position on deck, KG 12.0m, 7.2m either side of the vessels centreline.
The inboard weight is to be loaded first.
Using the Stability Data Booklet, calculate the minimum initial GM.
Determine initial and KM using initial draught of 7.30m in FW
FW

= 14901 t

KM = 8.33m

Determine GM (worst condition)


Calculate Moments about the Centreline
Weights (t)
14901
42
42
14985
GG H

Tan 4
or, GM

Distance from C L (m)


0.00
7.2
17.5

Moments about the C L (tm)


0.00
302.4
735.0
1037.4

= 1037.4 = 0.069m
14985
= 0.069
GM
= 0.987m

Determine KG on completion of loading operations

KM
GM
KG

= 8.33m
= 0.987m
= 7.343m

(from hydrostatic particulars using = 14985 t)

Determine KG prior commencing loading operations


Calculate Moments about the Keel
Weights (t)
14985
(-) 42
(-) 42
14901

KG

KG (m)
7.343
12.0
26.5

Moments about the Keel (tm)


110034.855
(-) 504.00
(-) 1113.00
108417.855

= 108417.855 = 7.276m
14901

Determine initial GM
Initial KM
KG
Initial GM

= 8.33m
= 7.276m
= 1.054m

2. A vessel is planning to enter dry-dock in salt water.


Present draughts: Forward 4.00m Aft 5.60m
KG 8.74m
Using the Stability Data Booklet, calculate EACH of the following:

ARNAB SARKAR

SQA Stability March 2007


(a) The maximum trim at which the vessel can enter dry-dock so as to maintain a GM
of at least 0.15m at the critical instant.
Note: Assume KM remains constant
(b) The weight of ballast to transfer from the aft peak to the fore peak in order to
reduce the trim to the maximum allowable.
(a) Calculate TMD
Forward draught
Aft draught

= 4.000m
= 5.600m

AMD = 4.800m

&

trim = 1.600m by stern

Using AMD = 4.800m, from the hydrostatic particulars,


LCF
TMD

= 69.29m foap
= 5.600 (1.600 x 69.29) = 4.794m
137.5

Calculate , KM, MCTC & LCF from hydrostatic particulars using TMD = 4.794m

= 9420 + (218 x 0.094)


0.10
KM = 9.22 (0.09 x 0.094)
0.10
MCTC = 156.0 + (0.9 x 0.094)
0.10
LCF = 69.35 (0.06 x 0.094)
0.10

= 9624.92 t
= 9.135m
= 156.846 tm
= 69.294m foap

Calculate the P force


Initial GM
= 9.135 8.74 = 0.395m
Reqd. GM
= 0.15m
Loss of GM = 0.395 0.015 = 0.245m
0.245
or, P

= P x 9.135
9624.92
= 258.14 t

Calculate required trim during dry-docking


258.14 = trim x 156.846
69.294
Reqd. trim = 114.0cms or, 1.140m by stern

(b) Calculate the initial LCG of the vessel prior transferring ballast
Using TMD = 4.794m, from hydrostatic particulars,
Initial LCB

= 70.90 (0.04 x 0.094) = 70.862m foap


0.10
1.6 x 100 = 9624.92 x (70.862 LCG)
156.846
or, LCG Initial = 68.255m foap
Calculate the final LCG of the vessel after transfer of ballast to attain required trim
1.140 x 100

= 9624.92 x (70.862 LCG)


156.846

ARNAB SARKAR

SQA Stability March 2007


or, LCG Final

= 69.004m foap

Assume that w tonne of ballast is transferred from aft peak to fore peak tank in order to
attain the required trim.
Calculating longitudinal moments about the AP
Weight (t)
9624.92
() w
(+) w
9624.92

LCG (m)
68.255
3.07
130.56

Longitudinal Moments (tm)


656948.91
3.07 w
130.56 w
(656948.91 127.49 w)

69.004

= (656948.91 127.49 w)
9624.92
or, w
= 56.5 t
Quantity of ballast to be transferred in order to attain the required trim = 56.5 t

3. A vessels loaded particulars in salt water are as follows:


Displacement 18000 tonne
Fluid KG 8.10m

Using the Stability Data Booklet, sketch the vessels statical stability curve and compare
the vessels stability with ALL the minimum stability criteria required by the current
Load Line Regulations. Comment on the result.
3. = 18000 t

KG F = 8.10m

Determine Angle of Flooding ( F ) for = 18000 t


F

= 40.6

Derive the KN values from the Hydrostatic Particulars

Angle of Heel ()

(Tonnes)

12

20

30

40

50

60

75

18000

1.75

2.93

4.27

5.36

6.12

6.67

6.92

Calculate the GZ values


Heel ()

KN (m)

KG Sin (m)

GZ (m)

12
20
30
40
50
60
75

1.75
2.93
4.27
5.36
6.12
6.67
6.92

1.68
2.77
4.05
5.21
6.20
7.01
7.82

0.07
0.16
0.22
0.15
- 0.08
- 0.34
- 0.90

Calculate the vessels GM F


From hydrostatic particulars, using = 18000 t in SW
KM T = 8.39 + (0.02 x 122) = 8.40m
241

ARNAB SARKAR

SQA Stability March 2007


GM = 8.40 8.10

= 0.30m

Construct the GZ Curve


0.4
0.2
0
-0.2

20

40

60

80

-0.4
-0.6
-0.8
-1

Calculate the area under the GZ Curve from 0 30


Angle ()
0
10
20
30

GZ (m)
0
0.06
0.16
0.22

SM
1
3
3
1

Product
0
0.18
0.48
0.22
0.88

Area 0 30

= 3 x 10 x 0.88
= 0.058 m-rad
8
57.3
Calculate the area under the GZ Curve from 0 40
Angle ()
0
10
20
30
40

GZ (m)
0
0.06
0.16
0.22
0.15

SM
1
4
2
4
1

Product
0
0.24
0.32
0.88
0.15
1.59

Area 0 40

= 1 x 10 x 1.59
= 0.092 m-rad
3
57.3
Calculate the area under the GZ Curve from 30 40
Area 30 40

= 0.092 0.058 = 0.040 m-rad

Summary

Actual

L.L. Regulation

Complies or Not

Area 0 30

0.058 m-r

Not less than 0.055 m-r

Yes

Area 0 40

0.092 m-r

Not less than 0.09 m-r

Yes

Area 30 40

0.040 m-r

Not less than 0.03 m-r

Yes

0.22m

At least 0.20m

Yes

Max. GZ

ARNAB SARKAR

SQA Stability March 2007


Max. GZ Angle
GM F

30

Equal to or greater than 30

Yes

0.30m

Not less than 0.15m

Yes

ARNAB SARKAR

SQA Stability July 2007


1. A vessel is to transit a canal with a minimum clearance of 0.40m under a bridge, the
underside of which is 20.38m above the waterline.
Present draughts in fresh water: Forward 5.42m
Aft 6.56m
The aft mast is 39m foap and extends 27.10m above the keel.
The fore mast is 108m foap and extends 26.00m above the keel.
(Assume masts are perpendicular to the waterline throughout)
Using the Stability Data Booklet, calculate EACH of the following:
(a) The final draughts forward and aft in order to pass under the bridge with
minimum clearance;
(b) The minimum weight of ballast to load in order to pass under the bridge with
minimum clearance.
a) Air draught required = 20.38 0.40 = 19.98m
=
=
=

Foremast
26.000m
19.980m
6.020m

Trim at Masts

7.120 6.020 = 1.100m by stern

Correction for Draught F


Trim at Masts

LBP Dist. of Foremast from AP


Dist. of Foremast from AP Dist. of Aft mast from AP

Correction for Draught F

(137.5 108.0) x 1.100


(108.0 39.0)

Correction for Draught A


Trim at Masts

Dist. of Aft mast from AP


Dist. of Foremast from AP Dist. of Aft mast from AP

Correction for Draught A

39.0
x 1.100 = 0.622m
(108.0 39.0)

Required draught
Required correction
Draft at perpendiculars

=
=
=

Height above the keel


Air draught required
Draught at Masts

Aft Mast
27.100m
19.980m
7.120m

Correction for Draught F

= 0.470m

Correction for Draught A

Calculate required draft at perpendiculars


Foremast
6.020m
(-) 0.470m
5.550m

Aft Mast
7.120m
(+) 0.622m
7.742m

b) Calculate Initial Displacement


Initial trim
Initial AMD

= 6.560 5.420
= 1.140m by stern
= (6.560 + 5.420)
= 5.990m
2
With AMD, from hydrostatic tables,
LCF = 68.43m (0.04 x 0.09)
= 68.39m
0.10
= 5.993m
TMD = 6.560 (1.140 x 68.39)
137.5
Displacement TMDi = 11778 + (219 x 0.093)
= 11981.67 t 11982 t
0.10

ARNAB SARKAR

SQA Stability July 2007


Calculate Final Displacement
Final trim
Final AMD

= 7.742 5.550
= (7.742 + 5.550)
2

= 2.192m by stern
= 6.646m

With AMD, from hydrostatic tables,


LCF = 67.79 (0.11 x 0.046)
= 67.74m
0.10
TMD = 7.742 (2.192 x 67.74)
= 6.662m
137.5
Displacement TMDf = 13324 + (224 x 0.062)
0.10

= 13462.88 t 13463 t

Calculate minimum weight of ballast to load in order to achieve required trim


Ballast to load

= Displacement TMDf Displacement TMDi


= 13463 11982 = 1481 t

2. A vessel is floating upright in fresh water and is about to enter dry-dock.


The vessel particulars are:
KG 8.37m
Forward draught 4.89m
Aft draught 6.71m
(a) Using the Stability Data Booklet, calculate the vessels effective GM at the critical
instant.
(b) Describe the methods of improving the initial stability is the GM at the critical instant
is found to be inadequate.
(a) Calculate the TMD
AMD = 5.800m

&

Trim

= 1.820m by stern

With AMD = 5.800m, from hydrostatic particulars,


LCF = 68.57m foap
TMD = 6.710 (1.820 x 68.57)
137.5

= 5.802m

Determine initial , MCTC & LCF using TMD = 5.802m


Initial
MCTC
LCF

= 11559 + (219 x 0.002)


0.10
= 163.2 + (1.2 x 0.002)
0.10
= 68.57 (0.14 x 0.002)
0.10

Calculate the P force


P
= 1.820 x 100 x 163.22
68.57
Displacement at critical instant

= 11563.38 t
= 163.22 tm
= 68.57m foap

= 433.22 t
= 11563.38 433.22 = 11130.16 t

Calculate KM using displacement at critical instant


KM = 8.67 (0.04 x 6.16)
= 8.67m
218

ARNAB SARKAR

SQA Stability July 2007


Calculate the loss of GM
Loss of GM

= 433.22 x 8.67
11563.38

= 0.325m

Calculate GM at critical instant


Initial GM
= 8.67 8.37 = 0.30m
Loss of GM = 0.325m
GM at C.I. = 0.30 0.325 = () 0.025m

(b) The trim can be reduced by transferring water, fuel or ballast forward.
The initial GM can also be increased by reducing FSM, lowering weights, or by ballasting
low or empty tanks.
3. A box shaped vessel floating on an even keel in salt water has the following particulars:
Length 130.00m

Breadth 24.00m

Draught 4.98m

There is a midship watertight compartment of length 20.00m, height 7.20m that


extends the full width of the vessel and is filled with cargo of relative density 0.80
stowing at 1.70 m3/t.
If this compartment is bilged, calculate EACH of the following:
(a) The final draught;
(b) The change in GM.
(a) Calculate the bilged draught
Volume of the box shaped vessel before bilging
Volume of the bilged compartment
Solid factor

= 130 x 24 x 4.98
= 20 x 24 x 4.98

= 15537.6 m3
= 2390.4 m3

1
= 1.25
0.80
Permeability = 1.70 1.25
= 0.265
1.70
Sinkage
=
2390.4 x 0.265
= 0.212m
(130 x 24) (20 x 24 x 0.265)
Initial draught
= 4.98m
Final draught
= 4.98 + 0.212 = 5.192m

(b) Before bilging

= 4.98
= 2.49m
2
KM = 9.639 + 2.49 = 12.129m

BM

= 130 x (24)3
12 x 15537.6

KB

BM

= (130 20 x 0.265) x (24)3 = 9.246m


12 x 15537.6

KB

= 9.639m

After bilging

= 5.192
= 2.596m
2
KM = 9.246 + 2.596 = 11.842m

Change in KM
Change in GM

= 12.129 11.842
= 0.287m decrease

= 0.287m decrease

ARNAB SARKAR

SQA Stability July 2007


4. A vessel plans to depart from port at:
Displacement 19000 t

KG 7.80m

During the ensuing voyage the vessel will consume 420 t of fuel oil (KG 0.60m) and 35 t
of fresh water (KG 7.35m) from full tanks causing a free surface moment of 1721 tm.
Using the Maximum KG Table in the Stability Data Booklet, determine the stability
condition of the ship BOTH on departure and on arrival.
On departure
As per the Maximum KG Table in the Stability Data Booklet,
Displacement (t)

KG (m)

19000

7.93

On departure, the vessels KG is 7.80m, therefore, it complies with the intact stability
criteria as specified in the current Load Line Rules.
On Arrival
Calculate moments about the Keel to determine the final KG
Weights (t)
19000
() 420
() 35
FSM
18545
Final KG

KG (m)
7.80
0.60
7.35

()
()
(+)

Vertical Moments (tm)


148200
252
257.25
1721
149411.75

= 8.057m

As per the Maximum KG Table in the Stability Data Booklet,


Displacement (t)

KG (m)

19000

7.93

18545

18500

8.02

Maximum KG for 18545 t = 8.02 (0.09 x 45) = 8.01m


500
On arrival, the vessels KG is 8.057m, which is more than specified in the Maximum KG
Table, therefore it does not comply with the intact stability criteria as specified in the current
Load Line Rules.

ARNAB SARKAR

SQA Stability November 2007


1. A vessel is to load a cargo of grain (stowage factor 1.62 m3/t).
Initial displacement 6300 tonne.
Initial KG 6.50m.
All five holds are to be loaded full of grain.
The tween decks are to be loaded as follows:
No. 1 TD
No. 2 TD
No. 3 TD
No. 4 TD

Part full ullage 1.50m


Full
Full
Part full ullage 2.75m

The Stability Data Booklet provides the necessary cargo compartment data for the
vessel.
(a) Using the Maximum Permissible Grain Heeling Moment Table included in the Stability
Data Booklet, determine whether the vessel complies with the minimum criteria
specified in the International Grain Code (IMO).
(b) Calculate the ships approximate angle of heel in the event of the grain shifting as
assumed by the International Grain Code (IMO).
(a) Initial
Initial KG
SF

= 6300 t
= 6.50m
= 1.62 m3/t

Calculate the Final , KG & VHM


Comp.

Ullage
(m)

Volume
(m3)

SF
(m3/t)

Initial

Vert. Mom.
(tm)
40950

VHM
(tm)

6300

KG
(m)
6.50

Tonnes

No.1 TD

1.50

1510

1.62

932

10.94

10196

1176

No.2 TD

Full

1676

1.62

1035

10.78

11157

539

No.3 TD

Full

1626

1.62

1004

10.59

10632

578

No.4 TD

2.75

900

1.62

556

10.20

5671

3278

No.1 Hold

Full

2215

1.62

1367

5.09

6958

410

No.2 Hold

Full

4672

1.62

2884

4.95

14276

1285

No.3 Hold

Full

1742

1.62

1075

4.94

5311

475

No.4 Hold

Full

3474

1.62

2144

4.95

10613

910

No.5 Hold

Full

2605

1.62

1608

8.76

14086

455

18905

6.87

129850

9106

Final
Calculate AHM
AHM = 9106
1.62

= 5621 tm

Calculate Maximum Permissible Heeling Moment


= 18905 t

KG F = 6.87m

ARNAB SARKAR

SQA Stability November 2007


Interpolation

KG F = 6.80

KG F = 6.87

KG F = 6.90

19000

7577

7283.0

7157

18905

7233.2

18500

7020.7

7307

6898

Holding = 19000 t constant, using variable KG F = 6.87m,


7577 (420 x 0.07) = 7283.0 tm
0.10
Holding = 18500 t constant, using variable KG F = 6.87m,
7307 (409 x 0.07) = 7020.7 tm
0.10
Holding KG F = 6.87m constant, using variable = 18905 t,
7020.7 + (262.3 x 405)
500

= 7233.2 tm

PHM for = 18905 t & KG F = 6.87m = 7233.2 tm

As AHM < PHM, vessel complies with the minimum criteria.


(b) Calculate approximate angle of list in event of grain shift
Angle of Heel = 5621 x 12 = 9.3
7233.2
2. A box shaped vessel, length 90.00m, breadth 10.00m, depth 9.00m, is floating at a
draught of 4.10m in salt water. Initial KG 3.70m.
(a) Calculate the angle of loll if 580 tonne of cargo, KG 7.80m is loaded on deck.
(b) Calculate the GM at the angle of loll in Q2 (a).
(a) Calculate the displacement of the box shaped vessel
Initial

= 90.00 x 10.00 x 4.10 x 1.025

= 3782.25 t

Calculate moments about the Keel to determine the Final KG


Weights (t)
3782.25
580
4362.25
Final KG

KG (m)
3.70
7.80

Vertical Moments (tm)


13994.325
4524
18518.325

= 4.245m

Calculate the final draught of the box shaped vessel after loading the cargo
Final

= 4362.25
Final draught (d)

= 90.00 x 10.00 x d x 1.025


= 4.729m

Calculate the final KM


KB
BM

= 4.729
= 2.365m
2
=
90.00 x (10.00)3
12 x 90.00 x 10.00 x 4.729

= 1.762m

ARNAB SARKAR

SQA Stability November 2007


KM = 2.365 + 1.762 = 4.127m
Calculate the Final GM after loading the cargo
Final GM

= 4.127 4.245

= () 0.118m

Calculate Angle of Loll


Tan (Angle of Loll)
Angle of Loll

(b) GM at angle of Loll

= (2 x 0.118)
1.762
= 20.1
= 2 x 0.118
Cos 20.1

= 0.251m

3. A box shaped vessel floating on even keel in salt water has the following particulars:
Length
Draught

160.00m
9.10m

Breadth
KG

22.00m
6.60m

There is an empty forward end compartment, length 20.00m extending the full width
of the vessel.
Calculate the draughts forward and aft, if this compartment is bilged.
Calculate the Bilged TMD of the box shaped vessel
Volume of the vessel
Displacement of the vessel

= 160.00 x 22.00 x 9.10


= 32032 x 1.025

Volume of the bilged compartment = 20.00 x 22.00 x 9.10

Intact WPA
Sinkage

Initial draught of the vessel

Bilged TMD

= (160.00 20.00) x 22.00


= 1.300m
= 4004
3080
= 9.100m
= 9.100 + 1.300 = 10.400m

= 32032 m3
= 32832.8 t
= 4004 m3
= 3080 m2

Calculate the trimming moment


Horizontal shift of B

Trimming Moment

= 20 = 10.00m aft
2
= 10.00 x 32832.8
= 328328 tm

Calculate Bilged GM L
Bilged KB

= 10.400
2
Bilged BM L = (140)3 x 22
12 x 32032
Bilged KM L = 157.05 + 5.2
Bilged GM L = 162.25 6.6

= 5.200m
= 157.05m
= 162.25m
= 155.65m

Calculate COT

MCTC = 32832.8 x 155.65


100 x 160.0
COT = 328328
319.40
LCF = (160 20)
2

= 319.40 tm
= 1027.9cms or 10.279m
= 70.0m

ARNAB SARKAR

SQA Stability November 2007

COT A = 10.279 x 70.0


160
COT F = 10.279 4.497

= 4.497m
= 5.782m

Calculate the Final Draughts


Bilged TMD
COT F/A
New Draughts

Forward
10.400
(+) 5.782
16.182m

Aft
10.400
() 4.497
5.903m

ARNAB SARKAR

SQA Stability March 2008


1. A vessel of length 152.00m, KG 8.28m, is floating upright in salt water at a True Mean
Draught of 5.50m.
The vessel has a rectangular double bottom of length 22.00m, breadth 16.00m, depth
2.00m, which is subdivided by a longitudinal centreline division into port and
starboard tanks of equal dimensions.
Using the Stability Data Booklet, calculate the angle of heel after partially filling the
port side of this tank with 240 tonne of fuel oil, relative density 0.874.
Remarks: The length of the vessel though irrelevant is given wrong. It should be 137.5m
instead of 152.0m
1. Using TMD = 5.500m, from the hydrostatic particulars,
Initial

= 11180 t

Determine the depth of the liquid in the tank after partially filling it up with 240 t of Fuel Oil
Breadth of the entire DB tank
Breadth of each Port/Stbd tank

= 16.00m
= 8.00m

240
= 22 x 8 x d x 0.874
d
= 1.560m
KG F.O. = 0.780m

Determine FSM caused due to the Fuel Oil


FSM

= 22 x (8)3 x 0.874
12

= 820.39 tm

Calculate Moments about the Keel to determine the Final KG


Weights (t)
11180
240
FSM
11420
Final KG

KG (m)
8.28
0.78

Vertical Moments (tm)


92570.4
187.2
820.39
93577.99

= 8.194m

Fluid

Determine the Final KM using the Final = 11420, from the hydrostatic particulars
Final KM
Determine GM

= 8.67 (0.04 x 18)


223

= 8.67m

Fluid

GM Fluid

= 8.67 8.194 = 0.476m

Determine GG H
GG H

= 240 x 4
11420

Determine the Angle of Heel


Tan

= 0.084
0.476
= 10.0 to port

= 0.084m

ARNAB SARKAR

SQA Stability March 2008


2. A vessels present particulars are:
Forward draught 7.60m, Aft draught 8.80m in salt water.
A total of 300 tonne of cargo, LCG 74.00m foap, is to be discharged immediately and
then the vessel is to proceed to an upriver berth where the relative density of the dock
water is 1.007.
During the passage the following items of deadweight are consumed:
80 t of Heavy Fuel Oil from
22 t of Diesel Oil from
22 t of Diesel Oil from
15 t of Fresh Water from

No.3 DB Centre tank


No. 4 DB Port tank
No.4 DB Starboard tank
After Fresh Water tank

Using the Stability Data Booklet, calculate the draughts fore and aft, on arrival at the
upriver berth.
Determine TMD of the vessel
AMD = 8.200m

Trim = 1.200m by stern

From hydrostatic particulars, using AMD = 8.200m,

LCF = 66.17m foap


TMD = 8.800 (1.200 x 66.17)
137.5

= 8.223m

Determine initial LCG of the vessel by using TMD = 8.223m from the hydrostatic
particulars,

= 17399 + (240 x 0.023)


0.10
MCTC = 202.4 + (1.4 x 0.023)
0.10
LCB = 69.49 (0.04 x 0.023)
0.10

= 17454.2 t
= 202.7 tm
= 69.48m foap

As vessel is initially trimmed by the stern, therefore LCB > LCG.


1.200 x 100 = 17454.2 x (69.48 LCG)
202.7
or,
LCG = 68.09m foap
Determine Final LCG on completion of upriver passage
Calculate Longitudinal Moments about the AP
Weights (t)
LCG (m)
17454.2
68.09
() 300
74.00
() 80
57.02
() 22
35.66
() 22
35.50
() 15
28.67
17015.2
Final LCG = 68.16m foap

Longitudinal Moments (tm)


1188456.5
() 22200
() 4561.60
() 784.52
() 781.00
() 430.05
1159699.33

ARNAB SARKAR

SQA Stability March 2008


Calculate volume in FW using Final = 17015.2 t
Volume in FW = 17015.2
1.007
Using Volume

FW

= 16896.9 m3

= 16896.9 m3, from hydrostatic particulars,

TMD

= 8.10 + (0.10 x 154.9)


233

= 8.167m

MCTC FW

= 196.1 + (1.4 x 154.9)


233
= 197.0 x 1.007

= 197.0 tm

MCTC 1.007
LCB
LCF

= 198.4 tm

= 69.54 (0.05 x 154.9)


233
= 66.25 (0.08 x 154.9)
233

= 69.51 m foap
= 66.20 m foap

Determine COT
COT

= 17015.2 x (69.51 68.16) = 115.8cms or 1.158m by stern


198.4

Determine Final Draughts

COT A = 1.158 x 66.20


137.5
COT F = 1.158 0.558

TMD
COT F/A
Final Draughts

= 0.558m
= 0.600m

Forward
8.167m
0.600m
7.567m

Aft
8.167m
0.558m
8.725m

3. A vessels loaded particulars in salt water are as follows:


Displacement 15000 tonne

Fluid KG 8.10m

Using the Stability Data Booklet, compare the vessels stability with all the minimum
stability criteria required by the current Load Line Regulations.
3. = 15000 t

KG F = 8.10m

Determine Angle of Flooding ( F ) for = 15000 t


F

= 46.5

Derive the KN values from the Hydrostatic Particulars

(Tonnes)
15000

12
1.73

20
2.98

Angle of Heel ()
30
40
50
4.48
5.72
6.48

60
6.91

Calculate the GZ values


Heel ()
12
20
30

KN (m)
1.73
2.98
4.48

KG Sin (m)
1.68
2.77
4.05

GZ (m)
0.05
0.21
0.43

75
7.04

ARNAB SARKAR

SQA Stability March 2008


40
50
60
75

5.72
6.48
6.91
7.04

5.21
6.20
7.01
7.82

0.51
0.28
- 0.10
- 0.78

Calculate the vessels GM F


From hydrostatic particulars, using = 15000 t in SW
KM T = 8.34 - (0.01 x 192) = 8.33m
232
GM F = 8.33 8.10
= 0.23m
Construct the GZ Curve
0.6
0.4
0.2
0
-0.2

20

40

60

-0.4
-0.6
-0.8
-1

Calculate the area under the GZ Curve from 0 30


Angle ()
0
10
20
30

GZ (m)
0
0.04
0.21
0.43

SM
1
3
3
1

Product
0
0.12
0.63
0.43
1.18

Area 0 30

= 3 x 10 x 1.18
= 0.077 m-rad
8
57.3
Calculate the area under the GZ Curve from 0 40
Angle ()
0
10
20
30
40
Area 0 40

= 1 x 10 x 2.81
3
57.3

GZ (m)
0
0.04
0.21
0.43
0.51

SM
1
4
2
4
1

= 0.163 m-rad

Product
0
0.16
0.42
1.72
0.51
2.81

80

ARNAB SARKAR

SQA Stability March 2008


Calculate the area under the GZ Curve from 30 40
Area 30 40

= 0.163 0.077 = 0.086 m-rad

Summary
Actual

L.L. Regulation

Complies or Not

Area 0 30

0.077 m-r

Not less than 0.055 m-r

Yes

Area 0 40

0.163 m-r

Not less than 0.09 m-r

Yes

Area 30 40

0.086 m-r

Not less than 0.03 m-r

Yes

0.51m

At least 0.20m

Yes

40

Equal to or greater than 30

Yes

0.23m

Not less than 0.15m

Yes

Max. GZ
Max. GZ Angle
GM F

ARNAB SARKAR

SQA Stability July 2008


1. A vessels present particulars are as follows:
A vessel is floating in salt water at a Forward draught of 8.300m and Aft draught of
8.500m. KG 8.00m. Vessel upright.
The vessel is to load bunkers of heavy fuel oil into No.3 DB port and starboard tanks
and sail upright at a maximum draught of 8.500m.
Using the Stability Data Booklet, calculate EACH of the following:
(a) The maximum weight of bunkers to load.
(b) The weight of ballast to transfer between the Aft Peak and the Fore Peak so that
the vessel sails on an even keel.
(a) Calculate TMD of the vessel
AMD = 8.400m

Trim = 0.200m by the stern

From hydrostatic particulars, using AMD = 8.400m,


LCF

= 66.02m foap

TMD

= 8.500 (0.200 x 66.02) = 8.404m


137.5

Initial

= 17878 + (241 x 0.004)


0.10
= 205.1 + (1.3 x 0.004)
0.10
= 69.40 (0.04 x 0.004)
0.10

Calculate Initial , MCTC and LCB from hydrostatic particulars using TMD = 8.404m

MCTC
LCB

= 17887.64 t
= 205.2 tm
= 69.40m foap

Calculate the Initial LCG


0.200 x 100
LCG

= 17887.64 x (69.40 LCG)


205.2
= 69.17m foap

From hydrostatic particulars determine Final , MCTC and LCB using sailing draught of
8.500m E.K.
Final
MCTC
LCB

= 18119 t
= 206.4 tm
= 69.36m foap

Determine maximum weight of bunkers to load


Maximum weight of bunkers to load

= 18119 17887.64 = 231.36 t

(b) Assume that w tonne of ballast is required to be transferred from aft peak to fore peak
tank.
Calculating Longitudinal Moments about the AP
Weight (t)
17887.64
231.36
() w
(+) w
18119

LCG (m)
69.17
57.87
3.07
130.56

Longitudinal Moments (tm)


1237288.06
13388.8
() 3.07w
(+) 130.56
1250676.86 + 127.49 w

ARNAB SARKAR

SQA Stability July 2008


Determine LCB from hydrostatic particulars using TMD = 8.500m
LCB

= 69.36m foap

As vessel will sail out on an even keel draught of 8.500m,


therefore LCB = LCG = 69.36m foap
69.36
or,

= 1250676.86 + 127.49 w
18119
= 47.5 t of ballast

2. Worksheet Q2 Trim and Stability provides relevant to a particular condition of


loading in a vessel in salt water.
All holds and tween decks are to be filled with grain (S.F. 1.62 m3/t).
The Stability Data Booklet provides the necessary data for the vessel.
By completion of the Worksheet Q2 and showing any additional calculations in the
answer book, calculate EACH of the following:
(a) The effective metacentric height;
(b) The draughts forward and aft.
CONDITION: LOADED GRAIN S.F. 1.62 m3/t
Comp.

Grain
Capacity (m3)

Stowage
Factor
(m3/t)

Weight
(t)

KG
(m)

Vertical
Moment
(tm)

All holds

14708

1.62

9079

5.64

1 TD

1695

1.62

1046

2 TD

1676

1.62

3 TD

1626

4 TD

1674

Free
Surface
Moment
(tm)

LCG foap
(m)

Long. Moment (tm)

51206

69.1

627359

11.26

11778

115.5

120813

1035

10.78

11157

95.6

98946

1.62

1004

10.59

10632

74.1

74396

1.62

1033

10.57

10919

51.7

53406

Oil Fuel

913

1917

2184

FW

86

634

75

Lightship

3831

8.21

31453

61.7

236373

18027

7.32

131955

69.06

1244885

HYDROSTATICS

LBP = 137.5m

True Mean Draught =


8.40 + (0.10 x 149)
241
= 8.462m
MCTC =
205.1 + (1.3 x 149)
241
= 205.9 tm

31042

LCB foap =
69.40 (0.04 x 149)
241
= 69.38m
KM T =
8.39 + (0.02 x 149)
241
= 8.40m

LCF foap =
66.02 (0.07 x 149)
241
= 65.98m
GM F =
8.40 7.32 = 1.08m

ARNAB SARKAR

SQA Stability July 2008


Trim between Perpendiculars: 18027 x (69.38 69.06) = 28.02 cm or 0.280m
205.9
COT A = 0.280 x 65.98 = 0.134m
137.5
COT F = 0.280 0.134 = 0.146m
Draughts:
Forward: 8.462 0.146 = 8.316m

Aft = 8.462 + 1.158 = 8.596m

3. A vessel is floating at an even keel draught of 8.600m in salt water. KG 7.40m.


A midship rectangular deck 29.00m long and extending the full breadth of the vessel is
flooded with salt water to a depth of 0.40m.
Height of deck above keel 11.75m.
Using the Stability Data Booklet, calculate EACH of the following:
(a) The fluid GM;
(b) The angle of loll.
(a) Determine the Initial of the vessel from hydrostatic particulars using TMD = 8.600m
Initial SW

= 18359 t

Determine the weight of salt water in the flooded deck


Weight of salt water = 29.00 x 20.42 x 0.40 x 1.025 = 242.8 t
Determine the FSM caused due to salt water ingress
= 29.00 x (20.42)3 x 1.025 = 21092 tm
12
Calculate moments about the keel to determine the Final KG
FSM

Weights (t)
18359
242.8
FSM
18601.8

KG (m)
7.40
(11.75 + 0.20)

Vertical Moments (tm)


135856.6
2901.46
21092
159850.06

Final KG

= 8.59m

Final KM

= 8.43 + (0.02 x 0.8) = 8.43m


242

Determine the Final KM from the Final

Determine the Final GM Fluid


GM Fluid

= 8.43 8.59 = () 0.16

(b) Determine the Final KB from the Final


KB

= 4.56 (0.05 x 0.8) = 4.56m


242

BM T

= 8.43 4.56

Tan

= (2 x 0.16)
3.87
Angle of Loll = 16.0

= 3.87m

ARNAB SARKAR

SQA Stability November 2008


1.(a)

A vessel, initially upright, is to carry out an inclining test.


Present displacement 5530 t. KM 10.75m.
Total weights on board during the experiment:
Ballast
Bunkers
Waters
Boiler Water
Inclining Weight

350 t
172 t
93 t
16 t
50 t

KG 3.52m
KG 4.01m
KG 4.46m
KG 4.19m
KG 8.60m

Tank Full
Free Surface Moment 897 tm
Free Surface Moment 916 tm
Free Surface Moment 115 tm

A deck crane, weight 21 t and still ashore, will be fitted on the vessel at a KG of 9.90m
at a later date.
The plumb lines have an effective vertical length 8.24m. The inclining weights are
shifted transversely 7.3m on each occasion and the mean horizontal deflection of the
plumb line is 0.68m
Calculate the vessels Lightship KG.
(b)

Explain why a vessels Lightship KG may change over a period of time.


(a) Displacement = 5530 t
Plumb length = 8.24m
Trans shift
= 7.3m

KM
Deflection

GM F = w x d x plum length
W x deflection

= 10.75m
= 0.68m

= 50 x 7.3 x 8.24
5530 x 0.68
= 0.80m

Therefore,

KM = 10.75m
GM F = 0.80m
KG F = 9.95m

Calculating moments about the Keel,


Inclined
(-) Ballast
(-) Bunkers
(-) FSM Bunkers
(-) Water
(-) FSM Water
(-) Inclining Weight
Lightship KG

Weights (t)

KG (m)

Vertical Moments (tm)

5530
(-) 350
(-) 172

9.95
3.52
4.01

(-) 93

4.46

(-) 50
4865 t

8.60

55023.5
1232.0
689.72
897.0
414.78
916
430.0
50444 tm

= 50444
4865

= 10.37 m

b) A vessels lightship KG changes over a period of time due to the following reasons:
Constant of the vessel keep changing due to accumulated sludge in fuel tanks, mud
and rust in ballast tanks (un-pumpables)
Various stores remaining unconsumed might add to the constant.
Any structural changes will affect the light ship KG and light ship displacement

ARNAB SARKAR

SQA Stability November 2008


Lightship KG for a passenger vessel will change considerably over a period of time
mainly because of the left over baggage etc. will accumulate over a period of time
and add to the constant considerably.
2.

A vessel is to load a cargo of grain (Stowage Factor 1.65 m3/t).


Initial displacement 6200 t
Initial KG 8.50m
All five holds are to be loaded full of grain.
The tween decks are to be loaded as follows:
No.1 TD
No.2 TD
No.3 TD
No.4 TD

Full
Empty
Part Full Ullage 1.50m
Full

The Stability Data Booklet provides the necessary cargo compartment data for the
vessel.
Using the KN tables to construct a GZ curve, determine whether the vessel complies
with the minimum criteria specified in the International Grain Code (IMO).
Initial
Initial KG
SF

= 6200 t
= 8.50m
= 1.65 m3/t

Calculate the Final , KG & VHM


Comp.

Ullage
(m)

Volume
(m3)

SF
(m3/t)

Initial
No.1 TD

Full

No.2 TD

Empty

No.3 TD

Vert. Mom.
(tm)
52700

VHM
(tm)

6200

KG
(m)
8.50

Tonnes

1695

1.65

1027

11.26

11564

352

1.50

1399

1.65

848

10.45

8862

1900

No.4 TD

Full

1674

1.65

1015

10.57

10729

604

No.1 Hold

Full

2215

1.65

1342

5.09

6831

410

No.2 Hold

Full

4672

1.65

2832

4.95

14018

1285

No.3 Hold

Full

1742

1.65

1056

4.94

5217

475

No.4 Hold

Full

3474

1.65

2105

4.95

10420

910

No.5 Hold

Full

2605

1.65

1579

8.76

13832

455

18004

7.45

134173

6391

Final

Using Final = 18004 t, determine KM from the hydrostatic tables


KM
GM F

= 8.39 (0.02 x 126) = 8.38m


241
= 8.38 7.45
= 0.93m

Determine the Angle of Flooding


Angle of Flooding

= 40.6

ARNAB SARKAR

SQA Stability November 2008


Determine the GZ values from the KN table
Heel ()

KN (m)

KG Sin (m)

GZ (m)

12

1.75

1.55

0.20

20

2.93

2.55

0.38

30

4.27

3.73

0.54

40

5.36

4.79

0.57

50

6.12

5.71

0.41

60

6.67

6.45

0.22

75

6.92

7.20

- 0.28

Determine 0 and 40
0

40

= 0.215 x 0.8

6391
1.65 x 18004

= 0.215m
= 0.172m

Draw the GZ Curve


F 40.6
0.90

Max Ords
between
curves

0.70

GZ (m)

0.95
0.50

GM
0.30

0.10

0.0
-0.10

10.0 12

20.0

30.0

37 40.0

50.0

Angle of heel

From the graph,


Angle of Heel after assumed grain shift
Initial GM

= 12
= 0.93m

Calculate the area (Residual Dynamical Stability) from 12 37


Angle ()
12.0
20.3
28.7
37.0

GZ (m)
0.00
0.18
0.34
0.40

SM
1
3
3
1

Product
0.00
0.54
1.02
0.40
1.96

60.0

70.0

ARNAB SARKAR

SQA Stability November 2008


Area 0 30

= 3 x 10 x 1.96
8
57.3

= 0.128 m-rad

Check if vessel complies with the International Grain Code


The vessel complies with International Grain Code Regulations provided she is upright on
sailing as
Angle of Heel after assumed grain shift
= 12 (complies)
Residual Dynamical Stability
= 0.128 m-rad (complies not less than
0.075 m-rad)
Initial GM
= 0.93m (complies not less than 0.3m)
3.

A vessel has the following particulars:


Displacement 15000 t
KG 7.88m

Even Keel draught 8.170m


KM 8.33m

Max. Breadth 20.80m


KB 4.15m

(a) Calculate the angle and direction of heel when turning to port in a circle of diameter
490m at a speed of 17.2 knots.
Note: Assume 1 nautical mile = 1852m, and g = 9.81m/sec2
(b) Calculate the new maximum draught during the turn in Q 3(a), assuming the midship
cross-section can be considered rectangular.
(a) BG

= KG KB
= 7.88 4.15 = 3.73m

Speed during turning (v)


Radius of turn

GM

= KM KG
= 8.33 7.88 = 0.45m

= 17.2 Knots = 17.2 x 1852 = 8.85 m/sec


3600
= 490 = 245m
2

Angle of heel during turning = tan = (8.85)2 x 3.73


9.81 x 245 x 0.45
or, = 15.1 to stbd.
(b) Draught when heeled = (8.170 x Cos 15.1) + (0.5 x 20.80 x Sin 15.1)
= 10.60m

ARNAB SARKAR

SQA Stability April 2009


1. A vessels present particulars are:
Forward draught 8.180m

Aft draught 9.420m at an upriver berth in fresh water.

The vessel is to proceed downriver to cross a sand bar at the river entrance where the
relative density of the water is 1.020.
During the river passage the following items of deadweight will be consumed:
60 t
21 t
21 t
18 t

of Heavy Fuel Oil from


of Diesel Oil from
of Diesel Oil from
of Fresh Water from

No. 3 DB Centre tank


No. 4 DB Port tank
No. 4 DB Starboard tank
After Fresh Water tank

(a) Using the Stability Data Booklet, calculate the clearance of the vessel over the bar if the
depth of water over the sand bar is 9.500m.
(b) State the maximum clearance over the sand bar if the vessel is brought to an even keel
condition by internal transfer of ballast.
(a) Calculate TMD
AMD = 8.800m

Trim

= 1.24m by the stern

From hydrostatic particulars, using AMD = 8.800m


LCF
TMD

= 65.74m foap
= 9.420 (1.24 x 65.74) = 8.827m
137.5
Determine Initial , MCTC and LCB using TMD = 8.827m
Initial FW
MCTC FW
LCB

= 18383 + (237 x 0.027)


0.10
= 205.1 + (1.2 x 0.027)
0.10
= 69.22 (0.04 x 0.027)
0.10

= 18447 t
= 205.4 tm
= 69.21m foap

Determine Initial LCG


1.24 x 100

LCG

= 18447 x (69.21 LCG)


205.4
= 67.83m foap

Calculate moments about the AP to determine the Final and Final LCG
Weights (t)
18447
() 60
() 21
() 21
() 18
18327
Final FW
Final LCG

LCG (m)
67.83
57.02
35.66
35.50
28.67

Longitudinal Moments (tm)


1251260.01
3421.2
748.86
745.5
516.06
1245828.39

= 18327 t
= 67.98m foap

Determine volume in dock water density (1.020)


Volume

= 18327
1.020

= 17967.65 m3

ARNAB SARKAR

SQA Stability April 2009


Determine the TMD, MCTC, LCB and LCF using FW as 17967.65 t
TMD = 8.60 + (0.10 x 56.65)
236

= 8.624m

MCTC = 202.6 + (0.3 x 56.65)


= 202.7 tm
236
MCTC 1.020 = 202.7 x 1.020 = 206.8 tm
LCB

= 69.31 (0.04 x 56.65)


236

= 69.30m foap

LCF

= 65.87 (0.06 x 56.65)


236

= 65.86m foap

Determine COT
COT

= 18327 x (69.30 67.98)


206.8
COT A
= 1.170 x 65.86
137.5
COT F
= 1.170 0.560

= 117.0cms or 1.170m by stern (LCB > LCG)


= 0.560m
= 0.610m

Determine Final Draughts


TMD
COT F/A
Final Draughts

Forward
8.624m
0.610m
8.014m

Aft
8.624m
0.560m
9.184m

Clearance of the vessel over the bar

= 9.500 9.184 =0.316m

(b) Max. clearance over the sand bar if the vessel is in EK draught = 9.500 8.624 = 0.876m
2. A vessel is floating upright in salt water and is about to dry-dock.
The vessels particulars are:
Length 137.5m

KG 8.34m

Present draughts:
Forward 5.420m

Aft 6.560m

(a) Using the Hydrostatic Particulars included in the Stability Data Booklet, calculate the
vessels effective GM at the critical instant.
(b) Describe the methods of improving the initial stability if the GM at the critical instant
is found to be inadequate.
(a) Calculate Initial TMD
Initial Trim
Initial AMD

= 5.420 6.560
= (5.420 + 6.560)
2
With AMD, from hydrostatic tables,
LCF = 68.43 (0.04 x 0.09)
0.10
TMD = 6.560 (1.140 x 68.39)
137.5

= 1.140m
= 5.990m

= 68.39m foap
= 5.993m

ARNAB SARKAR

SQA Stability April 2009


Calculate Initial , MCTC, LCF
With TMD, from hydrostatic tables,

= 12073 + (224 x 0.093)


0.10
MCTC = 168.5 + (1.3 x 0.093)
0.10
LCF = 68.43 (0.04 x 0.093)
0.10

= 12281.32 t
= 169.7tm
= 68.39m

Calculate the P Force


P Force

= 1.140 x 100 x 169.7 = 282.87 t


68.39

Calculate Virtual
Virtual

= 12281.32 282.87 = 11998.45 t

Calculate the KM T using the virtual displacement


KM T = 8.59 (0.04 x 150.45)
225

= 8.56m

Calculate GM at C.I.
Initial GM
Loss of GM

= 8.56 8.34 = 0.22m


= 282.87 x 8.56
= 0.197m
12281.32
GM at C.I. = 0.22 0.197 = 0.023m

(b) The trim can be reduced by transferring water, fuel or ballast forward.
The initial GM can also be increased by reducing FSM, lowering weights, or by ballasting
low or empty tanks.
3. A vessel is loading in dock water of R.D. 1.013.
Present draughts:
Forward 7.940m
Midship (Port) 8.610m

Midship (Starboard) 8.550m

Aft 9.120m

The draught marks are displaced as follows:


Forward: 1.66m aft of FP
Aft: 2.14m forward of the AP
The midship marks are not displaced.
The Stability Data Booklet provides the necessary hydrostatic data for the vessel.
By completion of the Worksheet Q3 and showing any additional calculations in the
answer book, determine the vessels displacement.
Draught forward
Trim
FP Correction
Draught at FP

= 7.940m
= 9.120 7.940
= 1.66 x 1.180
137.5
= 7.940 0.014

= 1.180m by stern
= 0.014m
= 7.926m

ARNAB SARKAR

SQA Stability April 2009


Draught aft
AP Correction
Draught at AP
True Trim

= 9.120m
= 2.14 x 1.180
137.5
= 9.120 + 0.018
= 9.138 7.926

Draught (M) Port


Draught (M) Stbd

= 8.610m
= 8.550m

Draught (M) Mean

= 8.580m

= 0.018m
= 9.138m
= 1.212m by stern

Amidships Line Corr. = Nil


Draught at Midships = 8.580m
Corr. (M) draught = 7.926 + (6 x 8.580) + 9.138
8

= 8.568m

From hydrostatic particulars using draught 8.568m


TPC

= 24.08 + (0.05 x 0.068)


= 24.11tm
0.10
LCF
= 65.95 (0.08 x 0.068)
= 65.90m foap
0.10
Displacement
= 18119 + (240 x 0.068)
= 18282.2 t
0.10
Distance of LCF from Midships
= 68.75 65.90
= 2.85m
1st Trim Correction
= 2.85 x 121.2 x 24.11
= 60.568 t
137.5
MCTC 8.068m
MCTC 9.068m
2nd Trim Correction
Corrected
Dock Water

= 199.6 + (1.4 x 0.068)


= 200.6 tm
0.10
= 212.7 + (0.3 x 0.068)
= 212.9 tm
0.10
= 50 x (1.212)2 x 12.3
= 6.570 t
137.5
= 18282.2 + 60.568 + 6.570
= 18349.338 x 1.013
1.025

= 18349.338 t
= 18134.52 t

ARNAB SARKAR

SQA Stability July 2009


1. A box shaped vessel floating on an even keel in salt water has the following particulars:
Length 140.00m

Breadth 30.00m

Draught 6.500m

KG 9.80m

The vessel has a centreline watertight bulkhead with an empty amidship side
compartment of length 22.00m on each side of the vessel.
Calculate the angle of heel if ONE of these side compartments is bilged.
1. Calculate Sinkage and Bilged Draught
Volume of Buoyancy lost
or, (22 x 15 x 6.500)
or, Sinkage

=
=
=

Bilged draught

Volume of Buoyancy gained


{(140 x 30) (22 x 15)} x Sinkage
2145
= 0.554m
3870
6.500 + 0.554 = 7.054m

Calculate new location of LCF from the side XX


Calculate moments of area about the axis XX
Total Area
Bilged Area

Area (m2)
140 x 30 = 4200
22 x 15 = 330
3870

New location of LCF


Distance BB H

Distance from axis XX (m)


15.0
7.5
()

Moments (m4)
63000
2475
60525

= 15.640m
= 15.640 15.0 = 0.640m

Calculate Moment of Inertia (I) about the new LCF


= I XX Ad2
3
3
2
LL = [(140 x 30 ) (22 x 15 )] [{(140 x 30) (22 x 15)} x 15.640 ]
3
3
or, I LL = (1260000 24750) 946639.152
or, I LL = 288610.848 m4
I LL
or, I

Calculate bilged BM, KB, KM and GM


BM

288610.848
(140 x 30 x 6.500)
KB
= 7.054
2
KM = 10.572 + 3.527
GM = 14.099 9.800

= 10.572m
= 3.527m
= 14.099m
= 4.299m

Calculate List

Tan = 0.640
4.299
List = 8.5

2. A vessel in salt water is trimmed 0.32m by the stern and heeled 8 to starboard and has
the following particulars:
Displacement 10600 tonne

KG 8.52m

Tunnel side tanks, port and starboard, are full of fuel oil and have centroids 4.30m
each side of the centreline. No.3 DB tanks, port and starboard are empty and have
centroids 4.80m each side of the centreline.

ARNAB SARKAR

SQA Stability July 2009


Using the Stability Data Booklet, calculate EACH of the following:
(a) The total weight of fuel oil to transfer from the Tunnel side tanks to No.3 DB tanks in
order to bring the vessel to even keel.
(b) The final weight of fuel oil transferred into EACH of No.3 DB tanks in order to bring
the vessel upright, assuming equal quantities are taken from the Tunnel side port and
starboard tanks.
(a) Calculate initial MCTC of the vessel
MCTC = 160.8 + (1.0 x 84) = 161.2 tm
221
Calculate the initial trimming moment of the vessel
Initial trimming moment

= 0.32 x 100 x 161.2 = 5158.4 tm

Assume that w tonne of fuel oil is needed to be transferred from the Tunnel side tanks to
No.3 DB tanks in order to bring the vessel upright.

or,

5158.4 = w (57.87 21.08)


w
= 140.2 t

Total amount of fuel oil transferred to bring the vessel upright


Amount to be transferred from each Tunnel side tank

= 140.2 t
= 70.1 t

Calculate initial KM of the vessel


KM

= 8.86 (0.05 x 84)


221

= 8.84m

GM = 8.84 8.52

= 0.32m

Initial listing moment

= 10600 x 0.32 x Tan 8 = 476.7 tm to starboard

Calculate the initial listing moment of the vessel

Assume that w tonne of fuel oil is needed to be transferred from No.3 DB tank (Stbd) to
No.3 DB tank (Port) in order to bring the vessel upright.

or,

476.7 = w x (4.8 + 4.8)


w
= 49.7 t

Fuel oil transferred from No.3 DB (Stbd) to No.3 DB (Port)


= 49.7 t
Final quantity of Fuel oil transferred in No.3 DB (Stbd) = 70.1 49.7 = 20.4 t
Final quantity of Fuel oil transferred in No.3 DB (Port) = 70.1 + 49.7 = 119.8 t

3. A vessel is to enter dry-dock in dock water of relative density 1.013.


Drafts:

Forward 3.100m

Aft 4.700m

KG 9.55m

(a) Using the Stability Data Booklet, calculate the maximum trim at which the vessel can
enter dry-dock so as to maintain a GM of at least 0.15m at the critical instant.
Assume KM remains constant.
(b) Describe the measures to be taken to ensure that the stability of the vessel is adequate
when the dock is flooded prior to the ship leaving the dock.
(a) Determine the TMD of the vessel
AMD = 3.900m

&

Trim

= 1.600m by the stern

ARNAB SARKAR

SQA Stability July 2009


From hydrostatic particulars, using AMD = 3.900m
LCF = 69.88m foap
TMD = 4.700 (69.88 x 1.600)
137.5

= 3.887m

From hydrostatic particulars, using TMD = 3.887m, determine Initial , MCTC, LCF & KM
Initial
MCTC
LCF
KM

= {7296 + (209 x 0.087)} x 1.013


0.10
= {144.2 + (0.9 x 0.087)} x 1.013
0.10
= 69.94 (0.06 x 0.087)
0.10
= 10.25 (0.14 x 0.087)
0.10

Determine Loss of GM
Initial GM
= 10.13 9.55 = 0.58m
Required GM = 0.15m
Loss of GM = 0.58 0.15 = 0.43m
Determine the P force
0.43
P

= P x 10.13
7575.04
= 321.55 t

Determine required trim to enter the dry-dock


321.55 = trim x 146.9
69.89
Trim = 152.9cm or 1.529m

= 7575.04 t
= 146.9 tm
= 69.89m foap
= 10.13m

ARNAB SARKAR

SQA Stability November 2009


1. A box-shaped vessel floating on an even keel in salt water has the following
particulars:
Length 120.00m

Breadth 22.00m

Draught 5.00m

There is a midship compartment of length 16.00m, extending the full breadth of the
vessel and from the watertight tank top to the freeboard deck.
Permeability of compartment 0.65. Height of watertight tank top 1.80m.
If this compartment is bilged, calculate EACH of the following:
a) The final draft
b) The change in GM
a) Calculate Sinkage & Final draft of the box shaped vessel
Volume of the box-shaped vessel before bilging
Volume of the bilged compartment

= 120 x 22 x 5.000 = 13200 m3


= 16 x 22 x (5.000 1.800) = 1126.4 m3

Permeability of the bilged compartment

= 0.65

Intact Water-Plane Area

= (120 x 22) (16 x 22 x 0.65)


= 2411.2 m2

Sinkage caused due to bilging

= 1126.4 x 0.65 = 0.304m


2411.2
= 5.000 + 0.304 = 5.304m

Final draft of the vessel after bilging

b) Before Bilging
KB

= 2.50m

BM

= 120 x (22)3 = 8.067m


12 x 13200

KM = 2.50 + 8.067 =10.567m


After Bilging

Calculate KB by taking moments of volume about the Keel


Volume (m3)
120 x 22 x 5.304
() 16 x 22 x (5.304 1.800) x 0.65
13200.84

KB (m)
2.652
3.552

Moments (m4)
37134.79
() 2847.7
34287.09

KB = 2.597m

= {120 (16 x 0.65)} x 223 = 7.368m


12 x 13200
KM = 2.597 + 7.368 = 9.965m
BM

Change in KM
Change in GM

= 10.567 9.965 = 0.602m decrease


= 0.602m decrease

2. A box shaped vessel, length 96.00m, breadth 11.00m, is floating at a draft of 4.20m in
salt water. Initial KG 4.18m.
a) Calculate the angle of loll if 620 t of cargo, KG 7.78m, is loaded on deck.
b) Calculate the GM at the angle of loll in Q 2(a).

ARNAB SARKAR

SQA Stability November 2009


a) Calculate Initial of the box shaped vessel
Initial

= 96.00 x 11.00 x 4.20 x 1.025

= 4546.08 t

Determine final KG of the box shaped vessel after loading cargo


Weights (t)
4546.08
620
5166.08

KG (m)
4.18
7.78

Moments (tm)
19002.61
4823.6
23826.21

Final KG

= 23826.21
5166.08

5166.08

= 96.00 x 11.00 x d x 1.025


= 4.773m

= 4.612m

Calculate the final draught after loading cargo

Calculate final KM of the box shaped vessel


KB
BM

KM

= 2.387m
= 96 x (11)3
= 2.113m
12 x 96 x 11 x 4.773
= 2.387 + 2.113
= 4.500m

Calculate final GM of the box shaped vessel


Final GM

= 4.500 4.612

= () 0.112m

Calculate Angle of Loll ()

Tan = 2 x 0.112
2.113

= 18.0

b) GM at angle of loll

= 2 x 0.112
Cos 18.0

= 0.236m

3. Worksheet Q3 Trim and Stability provides data relevant to a particular condition of


loading in a vessel in salt water.
The Hydrostatic Particulars included in the Stability Data Booklet provides the
necessary hydrostatic data for the vessel.
By completion of the Worksheet Q3 and showing any additional calculations in the
answer book, calculate EACH of the following:
a) The effective metacentric height
b) The draughts forward and aft

ARNAB SARKAR

SQA Stability November 2009


CONDITION: LOADED GENERAL CARGO

Length of vessel: 137.50m

Comp.

Grain
Capacity (m3)

Stowage
Factor
(m3/t)

Weight
(t)

KG
(m)

Vertical
Moment
(tm)

All holds

14606

1.88

7769

5.86

1 TD

1365

2.22

615

2 TD

1332

2.36

3 TD

1435

4 TD

1230

Free
Surface
Moment
(tm)

LCG foap
(m)

Long. Moment (tm)

45526

68.6

532953

10.98

6753

114.0

70110

564

10.50

5922

95.6

53918

2.14

671

10.37

6958

74.0

49654

2.28

540

10.28

5551

53.0

28620

Oil Fuel

856

1786

30074

FW

83

610

4590

Lightship

3831

8.21

31453

69.68

266944

14929

7.00

104559

69.45

1036863

LCB foap =
LCF foap =
True Mean Draught =
69.99 (0.05 x 121)
67.24 (0.11 x 121)
7.10 + (0.10 x 121)
HYDROSTATICS
232
232
232
= 7.152m
= 69.96m
= 67.18m
MCTC =
KM T =
GM F =
186.2 + (1.6 x 121)
8.34 (0.01 x 121)
LBP = 137.5m
232
232
8.33 7.00 = 1.33m
= 187.0 tm
= 8.33m
Trim between Perpendiculars: 14929 x (69.96 69.45) = 40.72 cm or 0.407m by stern
187.0
COT A = 0.407 x 67.18 = 0.199m
137.5
COT F = 0.407 0.199 = 0.208m
Draughts:
Forward: 7.152 0.208 = 6.944m
Aft = 7.152 + 0.199 = 7.351m

ARNAB SARKAR

SQA Stability March 2010


1. A vessel, initially upright and on even keel, has the following particulars:
Draught (in salt water) 7.000m

Breadth 20.42m

KG 7.82m

Further particulars of the vessel can be found in the Stability Data Booklet.
The vessels heavy lift derrick is to be used to discharge a 58 tonne boiler from a
centreline position, KG 5.30m. The derrick head is 30.00m above the keel and 16.00m
from the ships centreline when plumbing overside.
a) Calculate the maximum list angle when the boiler is suspended by the derrick at its
maximum outreach during discharge.
b) Calculate the increase in draught when the vessel is at the maximum list angle
calculated in Q 1(a), assuming a rectangular cross section midships.
c) Calculate the maximum allowable KG prior to discharging the boiler in order to limit
the list angle to 5.
a) Determine the Initial of the vessel from hydrostatic particulars
Initial

= 14576 t

Calculate moments about the keel and the centreline to determine the Final KG & GG H
Weights (t)
14576
() 58
(+) 58
14576

KG (m)
7.82
5.30
30.0
7.918

Final KG
GG H

= 7.918m
= 0.064m

Moments (tm)
113984.32
() 307.4
(+) 1740.0
115416.92

Dist. From C L
16.0
0.064

Initial KM
= 8.340m (from hydrostatic particulars)
Initial GM
= 8.340 7.918 = 0.422m
Maximum list angle = Tan = 0.064
0.422
or, = 8.6
b) Draught when heeled = (7.000 x Cos 8.6) + (20.42 x Sin 8.6)
2
= 8.448m
Initial Draught
= 7.000m
Increase in draught = 8.448 7.000 = 1.448m
c) Determine the GM when listed to 5
Tan 5 = 0.064
GM

GM = 0.732m

Determine the KG when listed to 5


KM

= 8.340m

KG = 8.340 0.732 = 7.608m

Determine KG prior to commence operations


Weights (t)
14576
(+) 58
() 58
14576

KG (m)
7.608
5.30
30.0
7.510

Moments (tm)
110894.208
(+) 307.4
() 1740.0
109461.608

Moments
928
928

ARNAB SARKAR

SQA Stability March 2010


Required KG prior to commence operations = 7.510m

2. a) A vessel, initially upright, is to carry out an inclining test.


Present displacement 5300 t
KM 10.96m
Total weights on board during the experiment:
Ballast
390 t
KG 3.45m
Bunkers
175 t
KG 4.01m
Water
102 t
KG 4.45m
Boiler Water
20 t
KG 4.19m
Inclining weights
48 t
KG 8.42m

Tank full
Free Surface Moment 996 tm
Free Surface Moment 890 tm
Free Surface Moment 101 tm

A weather deck hatch cover, weight 20 t, ashore for repair will be fitted on the vessel at
a KG of 9.46m prior to sailing.
The plumblines have an effective vertical length of 8.00m. The inclining weights are
shifted transversely 7.60m on each occasion and the mean horizontal deflection of the
plumbline is 0.68m.
Calculate the vessels lightship KG.
b) List FIVE possible causes for a change to the vessels lightship KG over a period of
time.
(a) Displacement = 5300 t
Plumb length = 8.00m
Trans shift
= 7.6m

KM
Deflection

GM F = w x d x plum length
W x deflection

= 10.96m
= 0.68m

= 48 x 7.6 x 8.00
5300 x 0.68
= 0.81m

Therefore,

KM = 10.96m
GM F = 0.81m
KG F = 10.15m

Calculating moments about the Keel,


Inclined
() Ballast
() Bunkers
() FSM Bunkers
() Water
() FSM Water
() Inclining Weight
(+) Hatch Cover
Lightship KG

Weights (t)

KG (m)

Vertical Moments (tm)

5300
() 390
() 175

10.15
3.45
4.01

() 102

4.45

() 48
(+) 20
4605 t

8.60
9.46

53795
1345.5
701.75
996
453.9
890
404.16
189.2
49192.89 tm

= 49192.89
4605

= 10.682 m

b) A vessels lightship KG changes over a period of time due to the following reasons:
Constant of the vessel keep changing due to accumulated sludge in fuel oil tanks.
Constant of the vessel keep changing due to accumulated mud and rust in ballast
water tanks (un-pumpables).

ARNAB SARKAR

SQA Stability March 2010


Various stores remaining unconsumed might add to the constant.
Any structural changes will affect the light ship KG and light ship displacement
Lightship KG for a passenger vessel will change considerably over a period of time
mainly because of the left over baggage etc. will accumulate over a period of time
and add to the constant considerably.
3. A box shaped vessel floating on even keel in dock water of RD 1.015 has the following
particulars:
Length 130.00m
Draught 8.000m

Breadth 21.00m
MCTC (salt water) 300

There is an empty watertight forward end compartment, length 10.00m, height 6.70m,
extending the full width of the vessel.
Calculate the draughts forward and aft, if this compartment is bilged.
Calculate the Bilged TMD
Volume of the vessel before bilging
= 130 x 21 x 8
Displacement of the vessel before bilging = 21840 x 1.015

= 21840 m3
= 22167.6 t

Permeability of the bilged compartment

= 1.00

Volume of the forward end compartment


Intact water plane area

= 10 x 21 x 6.70
= 130 x 21

= 1407 m3
= 2730 m2

Sinkage caused to bilging

= 1407
2730
= 8.000 + 0.515

= 0.515m

Bilged TMD

= 8.515m

Calculate the Trimming Moment


Calculating moments of volume about the AP after bilging,
Volume (m3)
130 x 21 x 8.515
() 1407
21838.95

Distance from AP (m)


65.0
125.0

Moments (m4)
1510986.75
() 175875
1335111.75

B H = 61.134m
BB H = 65 61.134 = 3.866m
Trimming Moment = 22167.6 x 3.866 = 85699.94 tm by forward
Calculate COT after bilging
COT

= 85699.94 x 1.025 = 288.5cm or 2.885m


300 x 1.015
COT A
= 2.885 x 65
= 1.443m
130
COT F
= 2.885 1.443
= 1.442m
Calculate the Final Draughts of the vessel
Forward Draught
Aft Draught

= 8.515 + 1.442 = 9.957m


= 8.515 1.443 = 7.072m

ARNAB SARKAR

SQA Stability July 2010


1. A vessel is upright, starboard side alongside, at a draught of 6.00m in salt water. KG
8.30m.
There is a 27 t boiler on deck, KG 7.78m, which is to be discharged using vessels crane
the head of which is 26.0m above the keel. The boiler is to be lifted from a position on
the vessels centreline and landed on a railway truck ashore. The distance of railway
truck from vessels centreline is 23.30m.
Using the Hydrostatic Particulars included in the Stability Data Booklet, calculate
EACH of the following:
a) The maximum angle of heel during discharge;
b) The maximum angle of heel during discharge if the vessel was first listed 5 to port
prior to the discharge of the boiler.
a) Determine the Initial and KM of the vessel from hydrostatic particulars,
Initial

= 12297 t

KM

= 8.52m

Calculate moments about the Keel and the C L of the vessel,


Weights (t)

KG (m)

Moments (tm)

12297
() 27
(+) 27
12297

8.30
7.78
26.0

102065.1
210.06
702.0
102557.04

Final KG
GG H
Final GM

= 8.340m
= 0.051m to starboard
= 8.52 8.34 = 0.18m

Distance from C L
(P)
(S)
23.3
-

Moments (tm)
(P)
(S)
629.1
629.1

Determine the maximum angle of heel during discharge


Tan (angle of heel)
Angle of Heel

= 0.051
0.18
= 15.8 to starboard

b) Determine the GG H when initially listed to port 5,


GG H

= (8.52 8.30) x Tan 5

= 0.019m to port

Calculate moments about the Keel and the C L of the vessel,


Weights (t)

KG (m)

Moments (tm)

12297
() 27
(+) 27
12297

8.30
7.78
26.0

102065.1
210.06
702.0
102557.04

Final KG
GG H
Final GM

= 8.340m
= 0.032m to starboard
= 8.52 8.34 = 0.18m

Distance from C L
(P)
(S)
0.019
23.3
-

Determine the maximum angle of heel during discharge


Tan (angle of heel)
Angle of Heel

= 0.032
0.18
= 10.1 to starboard

Moments (tm)
(P)
(S)
233.64 629.1
395.46

ARNAB SARKAR

SQA Stability July 2010


2. A vessel, initially upright, is to carry out an inclining test.
Present displacement 4800 t, KM 10.58m.
Total weights on board during the experiment:
Ballast
Bunkers
Fresh Water
Inclining weights

400 t
175 t
85 t
52 t

KG 3.52m
KG 3.86m
KG 4.46m
KG 8.42m

Tank full
Free Surface Moments 997 tm
Free Surface Moments 810 tm

At the time of the experiment the boilers were empty. They would usually contain a
total of 24 t of water, KG 4.18m, with a free surface moment of 124 tm.
A deck crane, weight 18 t and still ashore will be fitted on the vessel at a KG of 9.85m
before the next cargo operation.
The plumbline has an effective vertical length of 7.88m. The inclining weights are
shifted transversely 7.20m on each occasion and the mean horizontal deflection of the
plumbline is 0.66m.
a) Calculate the vessels lightship KG.
b) Identify FIVE possible causes of a change in the vessels lightship KG over a period of
time.
(a) Displacement = 4800 t
Plumb length = 7.88m
Trans shift
= 7.20m

KM
Deflection

GM F = w x d x plum length
W x deflection

= 10.58m
= 0.66m

= 52 x 7.2 x 7.88
4800 x 0.66
= 0.93m

Therefore,

KM = 10.58m
GM F = 0.93m
KG F = 9.65m

Calculating moments about the Keel,


Inclined
() Ballast
() Bunkers
() FSM Bunkers
() Water
() FSM Water
() Inclining Weight
(+) Boiler Water
(+) FSM Boiler Water
(+) Hatch Cover
Lightship KG

Weights (t)

KG (m)

Vertical Moments (tm)

4800
() 400
() 175

9.65
3.52
3.86

() 85

4.46

() 52
(+) 24

8.42
4.18

(+) 18
4130 t

9.85

46320
1408
675.5
997
379.1
810
437.84
100.32
124
177.3
42014.18 tm

= 42014.18
4130

= 10.173 m

b) A vessels lightship KG changes over a period of time due to the following reasons:
Constant of the vessel keep changing due to accumulated sludge in fuel oil tanks.

ARNAB SARKAR

SQA Stability July 2010


Constant of the vessel keep changing due to accumulated mud and rust in ballast
water tanks (un-pumpables).
Various stores remaining unconsumed might add to the constant.
Any structural changes will affect the light ship KG and light ship displacement
Lightship KG for a passenger vessel will change considerably over a period of time
mainly because of the left over baggage etc. will accumulate over a period of time
and add to the constant considerably.
3. A vessel is to load a cargo of grain (stowage factor 1.62 m3/t).
Initial displacement 5900 t, Initial KG 6.65m
All five holds are to be loaded full of grain.
The tween decks are to be loaded as follows:
No. 1 TD
No. 2 TD
No. 3 TD
No. 4 TD

Part Full Ullage 1.75m


Full
Part Full Ullage 2.50m
Full

The Stability Data Booklet provides the necessary cargo compartment data for the
vessel.
a) Using the Maximum Permissible Grain Heeling Moment Table Included in the Stability
Data Booklet, determine whether the vessel complies with the minimum criteria
specified in the International Grain Code (IMO).
b) Calculate the ships approximate angle of heel in the event of the grain shifting as
assumed by the International Grain Code (IMO).
a) Initial
Initial KG
SF

= 5900 t
= 6.65m
= 1.62 m3/t

Calculate the Final , KG & VHM


Comp.

Ullage
(m)

Volume
(m3)

SF
(m3/t)

Initial

Vert. Mom.
(tm)
39235

VHM
(tm)

5900

KG
(m)
6.65

Tonnes

No.1 TD

1.75

1416

1.62

874

10.88

9509

1372

No.2 TD

Full

1676

1.62

1035

10.78

11157

539

No.3 TD

2.50

979

1.62

604

10.25

6191

3142

No.4 TD

Full

1674

1.62

1033

10.57

10919

604

No.1 Hold

Full

2215

1.62

1367

5.09

6958

410

No.2 Hold

Full

4672

1.62

2884

4.95

14276

1285

No.3 Hold

Full

1742

1.62

1075

4.94

5311

475

No.4 Hold

Full

3474

1.62

2144

4.95

10613

910

No.5 Hold

Full

2605

1.62

1608

8.76

14086

455

18524

6.92

128255

9192

Final

ARNAB SARKAR

SQA Stability July 2010


Calculate AHM
AHM = 9192
1.62

= 5674 tm

Calculate Maximum Permissible Heeling Moment


= 18524 t

KG F = 6.92m

Interpolation

KG F = 6.90

KG F = 6.92

KG F = 7.00

19000

7157

7070.0

6737

18524
18500

6828.4
6898

6816.2

Holding = 19000 t constant, using variable KG F = 6.92m,


7157 (420 x 0.02) = 7283.0 tm
0.10
Holding = 18500 t constant, using variable KG F = 6.92m,
6898 (409 x 0.02) = 7020.7 tm
0.10
Holding KG F = 6.92m constant, using variable = 18524 t,
6816.2 + (253.8 x 24) = 6828.4 tm
500
PHM for = 18524 t & KG F = 6.92m = 6828.4 tm

As AHM < PHM, vessel complies with the minimum criteria.


(b) Calculate approximate angle of list in event of grain shift
Angle of Heel = 5674 x 12 = 10.0
6828.4

6489

ARNAB SARKAR

SQA Stability December 2010


1. A vessel is floating in salt water at draughts 7.640m forward, 8.180m aft.
The vessel is to load a cargo as to finish on an even keel at a draught of 8.500m. Two
spaces available: No.1 hold and No.5 hold.
Using the Stability Data Booklet, calculate EACH of the following:
a) The total weight of cargo to load;
b) The weight of cargo to load in each compartment.
1. Calculate AMD and Trim
AMD = 7.910m

Trim

= 0.540m by stern

Calculate LCF and TMD


LCF

= 66.42 (0.09 x 0.01)


0.10
TMD = 8.180 (0.540 x 66.41)
137.5

= 66.41m
= 7.920m

Using Hydrostatic Particulars, calculate Initial , LCB, & MCTC


Initial
LCB
MCTC

= 16685 + (237 x 0.02)


0.10
= 69.63 (0.05 x 0.02)
0.10
= 198.2 (1.40 x 0.02)
0.10

= 16732.4 t
= 69.62m foap
= 197.9 tm

Calculate the Initial LCG


0.540 x 100
or, LCG

= 16732.4 x (69.62 LCG)


197.9
= 68.98m foap

Calculate the Final , from hydrostatic particulars using draught 8.500m


Final

= 18119 t

a) Calculate total amount of cargo to load


Total amount of cargo to load = 18119 16732.4 = 1386.6 t 1387 t
Assume w tons of cargo is loaded in No.1 Hold
Weight of cargo loaded in No.5 Hold
= (1387 w) t
Calculate Longitudinal Moments about the AP
Initial
No.1 Hold
No.5 Hold
Final

Weights (t)
16732.4
(+) w
(+) (1387 w)
18119

LCG (m)
68.98
114.48
8.76

(+)
(+)

Longitudinal Moments (tm)


1154200.952
114.48 w
12150.12 8.76w
(1154200.952 + 105.72w)

Final LCG = (1154200.952 + 105.72w) m foap


18119
Determine LCB from hydrostatic particulars, using draught 8.500m
LCB

= 69.36m foap

As vessel will be in even keel on completion of loading, hence LCB = LCG = 69.36m foap

ARNAB SARKAR

SQA Stability December 2010


(1154200.952 + 105.72w) = 69.36
18119
or,
w
= 969.9 t 970 t
b) Quantity of cargo to be loaded in No.1 Hold
Quantity of cargo to be loaded in No.5 Hold

= 970 t
= 417 t

2. A vessels loaded particulars in salt water are as follows:


Displacement 18000 tonne

Fluid KG 8.20m

Using the Stability Data Booklet, compare the vessels stability with ALL the minimum
stability criteria required by the current Load Line Regulations, commenting on the
result.
2. = 18000 t

KG F = 8.20m

Determine Angle of Flooding ( F ) for = 18000 t


F

= 40.6

Derive the KN values from the Hydrostatic Particulars

Angle of Heel ()

(Tonnes)

12

20

30

40

50

60

75

18000

1.75

2.93

4.27

5.36

6.12

6.67

6.92

Calculate the GZ values


Heel ()

KN (m)

KG Sin (m)

GZ (m)

12
20
30
40
50
60
75

1.75
2.93
4.27
5.36
6.12
6.67
6.92

1.70
2.80
4.10
5.27
6.28
7.10
7.92

0.05
0.13
0.17
0.09
- 0.16
- 0.43
- 1.00

Calculate the vessels GM F


From hydrostatic particulars, using = 18000 t in SW
KM T = 8.39 + (0.02 x 122) = 8.40m
241
GM = 8.40 8.20
= 0.20m

ARNAB SARKAR

SQA Stability December 2010


Construct the GZ Curve
0.4
0.2
0
-0.2 0

20

40

60

80

-0.4
-0.6
-0.8
-1
-1.2

Calculate the area under the GZ Curve from 0 30


Angle ()
0
10
20
30

GZ (m)
0
0.04
0.13
0.17

SM
1
3
3
1

Product
0
0.12
0.39
0.17
0.68

Area 0 30

= 3 x 10 x 0.68
= 0.045 m-rad
8
57.3
Calculate the area under the GZ Curve from 0 40
Angle ()
0
10
20
30
40

GZ (m)
0
0.04
0.13
0.17
0.09

SM
1
4
2
4
1

Product
0
0.16
0.26
0.68
0.09
1.19

Area 0 40

= 1 x 10 x 1.19
= 0.069 m-rad
3
57.3
Calculate the area under the GZ Curve from 30 40
Area 30 40

= 0.069 0.045 = 0.024 m-rad

Summary

Actual

L.L. Regulation

Complies or Not

Area 0 30

0.045 m-r

Not less than 0.055 m-r

No

Area 0 40

0.069 m-r

Not less than 0.09 m-r

No

Area 30 40

0.024 m-r

Not less than 0.03 m-r

No

0.17m

At least 0.20m

No

30

Equal to or greater than 30

Yes

0.20m

Not less than 0.15m

Yes

Max. GZ
Max. GZ Angle
GM F

ARNAB SARKAR

SQA Stability December 2010


Vessel does not comply with the minimum stability criteria as required by the current
Load Line Regulations.
3. A vessel is floating at an even keel draught of 7.80m in salt water. KG 7.14m.
A midship rectangular deck 29.00m long and extending the full breadth of the vessel is
flooded with saltwater to a depth of 0.60m.
Height of deck above keel 9.50m.
Using the Stability Data Booklet, calculate EACH of the following:
a) The fluid GM;
b) The angle of loll.
3. Calculate the vessels Initial from hydrostatic particulars using draught of 7.800m
Initial

= 16448 t

Calculate the amount of water ingress in the midship rectangular deck


Quantity of water ingress

= 29.00 x 20.42 x 0.60 x 1.025

= 364.2 t

Calculate the Final after the flooding


Final

= 16448 + 364.2 = 16812.2 t

Determine the Final KM & KB from hydrostatic particulars, using Final 16812.2 t
Final KM

= 8.35m

Final KB

= 4.12 + (0.05 x 127.2)


237

= 4.15m

Determine the FSM caused due to water ingress


FSM caused

= 29 x (20.42)3 x 1.025
12

= 21091.518 tm

Determine the Final KG by taking moments about the keel

(+)
(+)

Weights (t)
16448
364.2
FSM
16812.2

KG (m)
7.14
9.50

Final KG

= 141990.138 = 8.45m
16812.2

Final GM F

= 8.35 8.45 = 0.10m

a) Determine Final GM F

b) Determine Angle of Loll


BM T

= 8.35 4.15 = 4.20m

Angle of Loll = (2 x 0.10)


4.20

Loll = 12.3

Vertical Moments (tm)


117438.72
3459.9
21091.518
141990.138

ARNAB SARKAR

SQA Stability March 2011


1. A vessels present particulars are:
Forward draught 7.982m, aft draught 9.218m at an upriver berth in fresh water.
The vessel is to proceed downriver to cross a sand bar at the river entrance where the
relative density of the water is 1.018.
During the river passage the following items of deadweight will be consumed:
56 t
24 t
24 t
15 t

of Heavy Fuel Oil


of Diesel Oil
of Diesel Oil
of Fresh Water

from No.3 DB Centre tank


from No.4 DB Port tank
from No.4 DB Starboard tank
from Forward Fresh Water tank

a) Using the Stability Data Booklet, calculate the clearance of the vessel if the depth of
water over the sand bar is 9.440m.
b) State the maximum clearance over the sand bar if the vessel is brought to an even keel
condition by internal transfer of ballast.
a) Calculate AMD and Trim
AMD = 8.600m

Trim

= 1.236m by stern

Calculate LCF and TMD


LCF = 65.87m
TMD = 9.218 (1.236 x 65.87)
137.5

= 8.626m

Using Hydrostatic Particulars, calculate Initial , LCB, & MCTC


Initial
LCB
MCTC

= 17911 + (236 x 0.026)


0.10
= 69.31 (0.04 x 0.026)
0.10
= 202.6 (0.03 x 0.026)
0.10

= 17972.36 t
= 69.30m foap
= 202.6 tm

Calculate the Initial LCG


1.236 x 100
or, LCG

= 17972.36 x (69.30 LCG)


202.6
= 67.91m foap

Calculate Longitudinal Moments about the AP to determine the Final LCG


Initial
HFO
FSM HFO
DO
FSM DO
DO
FSM DO
FW
FSM FW
Final

Weights (t)
17972.36
() 56

LCG (m)
67.91
57.02

() 24

57.87

() 24

57.87

() 15

32.47

17853.36

()
(+)
()
(+)
()
(+)
()
(+)

Longitudinal Moments (tm)


1220502.968
3193.12
1162.556
1388.88
279.95
1388.88
279.95
487.05
29.522
1215797.016

ARNAB SARKAR

SQA Stability March 2011


Final LCG = 1215797.016
17853.36

= 68.10m foap

Volume 1.018 = 17853.36


1.018

= 17537.68

Determine volume using Final

From hydrostatic particulars, determine TMD, MCTC, LCB & LCF using Final
TMD
MCTC FW
MCTC 1.018
LCB

= 8.40 + (0.10 x 95.68)


235
= 200.1 + (1.3 x 95.68)
235
= 200.6 x 1.018
= 69.40 (0.04 x 95.68)
235
= 66.02 (0.07 x 95.68)
235

LCF

= 8.441m
= 200.6 tm
= 204.2 tm
= 69.38m foap
= 65.99m foap

Determine CoT
CoT

CoT A
CoT F

= 17853.36 x (69.38 68.10) = 111.9cm or 1.120m by stern


204.2
= 1.120 x 65.99
= 0.538m
137.5
= 1.120 0.538
= 0.582m

Determine Final Draughts


Draught Forward
Draught Aft

= 8.441 0.582 = 7.859m


= 8.441 + 0.538 = 8.979m

Determine the maximum clearance of the vessel over the sand bar
Clearance

= 9.440 8.979 = 0.461m

b) Calculate the new AMD and Trim with present draughts


AMD = 8.419m

Trim

= 1.120m by stern

Calculate LCF and TMD


LCF

= 66.02 (0.07 x 0.019)


0.10
TMD = 8.979 (1.120 x 66.01)
137.5

= 66.01m foap
= 8.441m

In even keel draught, vessels draught will be equal to TMD = 8.441m


Determine the maximum clearance of the vessel over the sand bar in Even Keel
Clearance

= 9.440 8.441 = 0.999m

2. A box shaped vessel floating on an even keel in salt water has the following particulars:
Length 100.00m
Draught 5.000m

Breadth 15.00m
KG 5.40m

Depth 9.00m

ARNAB SARKAR

SQA Stability March 2011


A midship watertight compartment 20.00m long and extending the full breadth and
depth of the vessel is bilged. Permeability of the compartment is 0.85.
Calculate EACH of the following:
a) The new draught;
b) The change in GM;
c) The righting moment in the flooded condition for an angle of heel of 20.
a) Volume of the box shaped vessel before bilging
Volume of the bilged compartment
Permeability of the bilged compartment

= 100 x 15 x 5 = 7500 m3
= 20 x 15 x 5 = 1500 m3
= 0.85

Calculate Sinkage and bilged draught


Sinkage

1500 x 0.85
(100 x 15) (20 x 15 x 0.85)

Bilged draught = 5.000 + 1.024

= 1.024m
= 6.024m

b) Determine the KM prior bilging


KB

= 5.000
= 2.500m
2
BM = 100 x 153 = 3.750m
12 x 7500
KM = 2.500 + 3.750 = 6.250m
Determine the KM after bilging

= 3.012m
= 6.024
2
BM = {100 (20 x 0.85) x 153
12 x 7500
KM = 3.012 + 3.113 = 6.125m
KB

= 3.113m

Determine change in GM
Change in KM
Change in GM

= 6.250 6.125 = 0.125m decrease


= 0.125m decrease

c) Calculate GZ
GM

= 6.125 5.400
= 7500 x 1.025

= 0.725m
= 7687.5 t

Using wall sided formula to determine GZ as the angle of heel is 20


GZ

= [0.725 + 3.113 x Tan2 20] Sin 20 = 0.318m


2

Righting Moment in flooded condition = 7687.5 x 0.318 = 2444.6 tm 2445 tm

3. A vessel completes under-deck loading in salt water with the following particulars:
Displacement 15040 tonne
KG 8.00m
The Stability Data Booklet provides the necessary data for the vessel.

ARNAB SARKAR

SQA Stability March 2011


During the passage, 200 tonne of Heavy Fuel Oil (RD 0.80) is consumed from No.3 DB
tank centre which was full on departure (use VCG of tank for consumption purposes).
Calculate the maximum weight of timber to load on deck KG 12.60m assuming 15%
water absorption during the passage in order to arrive at the destination with the
minimum GM (0.05m) allowed under the Load Line Regulations.
Note: Assume KM constant
3. Assume that w t of timber deck cargo is loaded on deck.
Absorption of water during the passage by the timber deck cargo

= 0.15 w t

Calculate the Initial KM of the vessel from hydrostatic particulars using = 15040 t
Initial KM

= 8.33m

Calculate the FSM for the HFO consumption


FSM HFO

= 1142 x 0.80 = 913.6 tm

Calculate the final KG of the vessel by calculating moments about the Keel

(+)
()
(+)

Weights (t)
15040
1.15w
200
FSM
(14840 + 1.15w)

Final KG

Final GM

KG (m)
8.00
12.6
0.60

Vertical Moments (tm)


120320
14.49w
()
120
(+)
913.6
(121113.6 + 14.49w)

= (121113.6 + 14.49w) m
(14840 + 1.15w)
= 8.33 (121113.6 + 14.49w)
(14840 + 1.15w)
or,
(121113.6 + 14.49w)
(14840 + 1.15w)
or,
121113.6 + 14.49w
or,
4.97w
or,
w

Total weight of timber deck cargo loaded

= 0.05
= 8.28
= 122875.2 + 9.52w
= 1761.6
= 354.4 t
= 354.4 t

ARNAB SARKAR

SQA Stability July 2011


1. A vessel is to transit a canal with a minimum clearance of 0.40m under a bridge, the
underside of which is 21.26m above the waterline.
Present draughts in fresh water (RD 1.000): Forward 5.380m Aft 6.560m
The fore mast is 110m foap and extends 26.00m above the keel.
The aft mast is 36m foap and extends 27.20m above the keel.
Using the Stability Data Booklet, calculate EACH of the following:
a) The final draughts forward and aft in order to pass under the bridge with minimum
clearance;
b) The maximum weight of cargo that can be discharged in order to pass under the
bridge with minimum clearance.
Note: Assume masts are perpendicular to the waterline throughout
a) Air draught required = 21.26 0.40 = 20.86m
=
=
=

Foremast
26.000m
20.860m
5.140m

Trim at Masts

6.340 5.140 = 1. 200m by stern

Correction for Draught F


Trim at Masts

LBP Dist. of Foremast from AP


Dist. of Foremast from AP Dist. of Aft mast from AP

Correction for Draught F

(137.5 110.0) x 1.200


(110.0 36.0)

Correction for Draught A


Trim at Masts

Dist. of Aft mast from AP


Dist. of Foremast from AP Dist. of Aft mast from AP

Correction for Draught A

36.0
x 1.200
(110.0 36.0)

Height above the keel


Air draught required
Draught at Masts

Aft Mast
27.200m
20.860m
6.340m

Correction for Draught F

= 0.446m

Correction for Draught A

= 0.584m

Calculate required draft at perpendiculars


Required draught
Required correction
Draft at perpendiculars

=
=
=

Foremast
5.140m
(-) 0.446m
4.694m

Aft Mast
6.340m
(+) 0.584m
6.924m

b) Calculate Initial Displacement


Initial trim
Initial AMD

= 6.560 5.380
= (6.560 + 5.380)
2
With AMD, from hydrostatic tables,
LCF = 68.43 (0.04 x 0.07)
0.10
TMD = 6.560 (1.180 x 68.40)
137.5

= 1.180m by stern
= 5.970m

= 68.40m foap
= 5.973m

ARNAB SARKAR

SQA Stability July 2011


Displacement

= 11778 + (219 x 0.073)


0.10
Calculate Final Displacement
Final trim
Final AMD

FW

= 6.924 4.694
= (6.924 + 4.694)
2

= 11937.87 t 11938 t

= 2.230m by stern
= 5.809m

With AMD, from hydrostatic tables,


LCF = 68.57 (0.14 x 0.009)
= 68.56m
0.10
TMD = 6.924 (2.230 x 68.56)
= 5.812m
137.5
Displacement FW

= 11559 + (219 x 0.012)


0.10

Weight of Cargo

= 11938 11585 = 353 t

= 11585.28 t 11585 t

Calculate maximum weight of cargo needed to be discharged in order to pass under bridge

2. A vessel entered drydock for emergency repairs while carrying cargo.


Particulars on entry (salt water):
Displacement 15716 t
Trim 0.64m by stern

KG 8.10m

While in drydock, 360 tonne of cargo, KG 3.80m, LCG 82.40m was discharged and is
no longer on board. The vessel is now planning to leave the drydock.
a) Using the Stability Data Booklet, calculate the vessels effective GM at the critical
instant on departure.
b) State the stability measures that should be considered prior to flooding the dock in
order to ensure the safe undocking of this vessel.
a) Calculate the Final KG after discharging the cargo

()

Weights (t)
15716
360
15356
Final KG

KG (m)
8.10
3.80
= 125931.6
15356

Vertical Moments (tm)


127299.6
1368
125931.6

= 8.201m

Determine vessels initial LCB & MCTC from hydrostatic particulars, using = 15716 t
LCB

= 69.85 (0.04 x 209)


235
MCTC = 190.9 + (1.5 x 209)
235

= 69.81m foap
= 192.2 tm

Calculate vessels initial LCG


0.64 x 100
or, LCG

= 15716 x (69.81 LCG)


192.2
= 69.03m foap

ARNAB SARKAR

SQA Stability July 2011


Calculate Final LCG of the vessel after discharging the cargo
Weights (t)
15716
360
15356

()

Final LCG

LCG (m)
69.03
82.40

Longitudinal Moments (tm)


1084875.48
29664.0
1055211.48

= 1055211.48 = 68.72m foap


15356

Determine vessels final LCB, LCF, MCTC & KM from hydrostatic particulars,
using = 15356 t
LCB

= 69.90 (0.05 x 82)


233
LCF = 67.03 + (0.11 x 82)
233
MCTC = 189.4 + (1.50 x 82)
233
KM = 8.33m

= 69.88m foap
= 67.07m foap
= 189.93 tm

Calculate vessels final trim on departure


Trim on departure

= 15356 x (69.88 68.72)


189.93

= 93.8cm or 0.938m

Calculate the P force


P

= 93.8 x 189.93
67.07

= 265.624 t

Calculate Loss of GM
Loss of GM

= 265.624 x 8.33
15356
Vessels virtual GM on departure
Vessels effective GM at C.I.

= 0.144m
= 8.33 8.201
= 0.129 0.144

= 0.129m
= 0.015m

b) The drydock cannot be flooded with the vessel in this condition as her GM is negative. It is
necessary to take adequate measures to ensure that the vessel has sufficient positive stability
at the critical instant during un-docking.
There are two options available (or a combination of both) to attain sufficient positive
stability prior to un-docking:
Reduce the vessels KG by taking in ballast or loading deadweight items below VCG
or remove deadweight items above VCG.
Reduce the vessels trim by removing deadweight items aft of the vessels LCG or
by adding deadweight items forward of the vessels LCG.
3. A vessel has the following particulars:
Even keel draught 8.720m
KG 7.96m
KM 8.38m
(i)
(ii)

Maximum breadth 22.60m


KB 4.18m

Calculate the angle and direction of heel when turning to port in a circle of
diameter 500m at a speed of 18.2 Knots.
Calculate the new maximum draught during the turn in Q 3(i), assuming the
midships cross-section can be considered rectangular.

ARNAB SARKAR

SQA Stability July 2011


Note: Assume 1 Nautical Mile = 1852m and g = 9.81m/sec2
(i)

Calculate BG and GM of the vessel


BG
GM

= 7.96 4.18 = 3.78m


= 8.38 7.96 = 0.42m

Calculate the radius and velocity of turn


R
v

= 500 = 250m
2
= 18.2 x 1852 = 9.363 m/sec2
3600

Calculate the angle and direction of heel when turning

(i)

Tan = (9.363)2 x 3.78


9.81 x 250 x 0.42

= 17.8 to stbd.
Calculate draught of the vessel when heeled

Draught when heeled = (8.720 x Cos 17.8) + (0.5 x 22.6 x Sin 17.8)
= 11.762m

ARNAB SARKAR

SQA Stability November 2011


1. As a result of a collision a vessel is listed 8 to port in salt water. A deep tank is
partially full of an oily water mixture, RD 0.95, which is to be fully discharged into a
salvage barge alongside. The deep tank is rectangular and is 14.0m long and 18.0m
wide.
Present displacement 16000 t
Weight of oily water mixture 846 t

KG 8.20m
KG 3.20m

Using Hydrostatic Particulars contained in the Stability Data Booklet, calculate EACH
of the following:
a) The final list of the vessel;
b) The weight of ballast to transfer from a port tank to a starboard tank in order to bring
the vessel upright. The centroid of EACH tank is 5.00m from the centreline.
a) Calculate the FSM
FSM

= 14.0 x (18.0)3 x 0.95


12 x 16000

= 6464 tm

Determine the Final KG by taking moments about the Keel,

()
(+)

Weights (t)
16000
846
FSM
15154

KG (m)
8.20
3.20

Final KG

= 122028.8
15154

Initial KM

= 8.33 + (0.01 x 24) = 8.33m


236
= 8.33 8.05
= 0.28m

Vertical Moments (tm)


131200
2707.20
6464
122028.8

= 8.05m

Calculate the Final GM after discharging the oily water mixture into the salvage barge

Final GM

(from hydrostatic particulars)

Calculate Initial Listing Moment of the vessel


Initial GM
Listing Moment

= 8.33 8.20
= 0.13m
= 16000 x 0.13 x Tan 8 = 292.3 tm to Port

Calculate the Final List


Tan = GG H
= x GG H
GM
x GM
Tan
= 292.3
15154 x 0.28

= 3.9 to Port

b) To bring the vessel to upright condition,


or,
or,

Port Moment = Stbd. Moment


292.3
= w x (5.00 + 5.00)
w
= 29.2 t

Weight of ballast to transfer to bring the vessel upright = 29.2 t

ARNAB SARKAR

SQA Stability November 2011


2. A box shaped vessel floating on an even keel in salt water has the following particulars:
Length 120.00m

Breadth 30.00m

Draught 6.600m

KG 11.50m

The vessel has a centreline watertight bulkhead with an empty amidships side
compartment of length 20.00m on each side of the vessel.
a) Calculate the angle of heel if ONE of these side compartments is bilged as a result of a
collision.
b) Calculate the clearance over a sand bar in the bilged condition at the entrance of a
port of refuge where the depth of water is 11.00m.
a) Calculate Sinkage & Bilged draught of the box shaped vessel
or,
or,

Volume of buoyancy lost


20.00 x 15.00 x 6.600
Sinkage
Sinkage
Bilged draught

= Volume of buoyancy gained


= {(120.00 x 30.00) (20.00 x 15.00)} x Sinkage
= 1980
3300
= 0.600m
= 6.600 + 0.600 = 7.200m

Calculate the new location of LCF with respect to axis XX


Calculating Moments of area about the axis XX

()

Area (m2)
120 x 30 = 3600
20 x 15 = 300
3300

Distance from axis XX (m)


15.0
7.50

Moments (tm)
54000
2250
51750

New Location of the LCF = 51750


= 15.682m
3300
= 15.682 15.000 = 0.682m
Distance BB H
Calculate Moment of Inertia about the axis XX
I

LL

= I XX Ad2
= {LB3 lb3} {(L x B) (l x b)} x d2
3
3
= {(120 x 303) (20 x 153)} {(120 x 30) (20 x 15)} x 15.6822
3
3
= 1057500 811552.909
= 245947.091 m4

Determine bilged KB, BM & KM


KB

= 7.200
2
BM = 245947.091
(120 x 30 x 6.60)
KM = 10.351 + 3.63

= 3.600m
= 10.351m
= 13.981m

Determine angle of heel after bilging


GM
Tan
or,

= 13.981 11.500
= 0.682
2.481
= 15.4

= 2.481m

ARNAB SARKAR

SQA Stability November 2011


b) Draught when heeled = (7.200 x Cos 15.4) + (0.5 x 30.0 x Sin 15.4)
= 10.925m
Clearance over sand bar = 11.000 10.925 = 0.075m

3. A vessel is floating upright in dock water of RD 1.012 and is about to dry dock.
The vessel particulars are:
Present draughts: Forward 5.340m

Aft 6.660m

KG 8.380m

Using the Stability Data Booklet, calculate the vessels effective GM at the critical
instant.
3. Calculate AMD and Trim
AMD = 6.000m

Trim

= 1.320m

Calculate LCF and TMD


LCF = 68.39m foap
TMD = 6.660 (1.320 x 68.39)
137.5

= 6.003m

Calculate Displacement, MCTC and LCF


Volume

FW

DW

MCTC FW
MCTC DW
LCF

= 11997 + (220 x 0.003) = 12004 m3


0.10
= 12004 x 1.012 = 12148 t
= 165.7 + (1.3 x 0.003) = 165.7 tm
0.10
= 165.7 x 1.012 = 167.7 tm
= 68.39 (0.09 x 0.003) = 68.39m foap
0.10

Calculate P Force and Virtual


P Force
Virtual

= 1.32 x 100 x 167.7 = 324 t


68.39
= 12148 324 = 11824 t

Calculate KM at C.I.
Virtual
= 11824 t
Volume DW = 11824
1.012

= 11684 t

= 11684 m3

Determine KM from hydrostatic particulars, using = 11684 t


KM

= 8.59 (0.04 x 125) = 8.57m


219

Calculate GM at C.I.
Loss of GM

= 324 x 8.57 = 0.23m


12148
GM
= 8.57 8.30 = 0.27m
GM at C.I. = 0.27 0.24 = 0.04m

ARNAB SARKAR

SQA Stability March 2012


1. Worksheet Q1 Trim and Stability provides data relevant to a particular condition of
loading in a vessel in salt water.
All holds and tween decks are to be filled with grain (SF 1.65m3/t).
The Stability Data Booklet provides the necessary data for the vessel.
By completion of the Worksheet Q1 and showing any additional calculations in the
answer book, calculate EACH of the following:
a) The effective metacentric height;
b) The drafts forward and aft.
CONDITION: LOADED GRAIN

Length of vessel: 137.50m

Comp.

Grain
Capacity (m3)

Stowage
Factor
(m3/t)

Weight
(t)

KG
(m)

Vertical
Moment
(tm)

All holds

14708

1.65

8914

5.64

1 TD

1695

1.65

1027

2 TD

1676

1.65

3 TD

1626

4 TD

1674

Free
Surface
Moment
(tm)

LCG foap
(m)

Long. Moment (tm)

50274.96

69.10

615957.40

11.26

11564.02

115.50

118618.50

1016

10.78

10952.48

95.60

97129.60

1.65

986

10.59

10441.74

74.10

73062.60

1.65

1015

10.57

10728.55

51.7

52475.50

Oil Fuel

934

1956

2218

31106

FW

83

629

73

2538

Lightship

3831

8.21

31452.51

61.7

236372.70

17806

7.317

130290.26

68.92

1227260.30

LCB foap =
True Mean Draught =
69.45 (0.05 x 167)
8.30 + (0.10 x 167)
HYDROSTATICS
239
239
= 8.370m
= 69.42m
MCTC =
KM T =
203.8 + (1.3 x 167)
8.38 + (0.01 x 167)
LBP = 137.5m
239
239
= 204.7 tm
= 8.39m
Trim between Perpendiculars: 17806 x (69.42 68.92) = 43.49 cm or 0.435m by stern
204.7
COT A = 0.435 x 66.04 = 0.209m
137.5
COT F = 0.435 0.209 = 0.226m
Draughts:
Forward: 8.370 0.226 = 8.144m

LCF foap =
66.10 (0.08 x 167)
239
= 66.04m
GM F =
8.39 7.317
= 1.073m

Aft = 8.370 + 0.209 = 8.579m

ARNAB SARKAR

SQA Stability March 2012


2. A vessel is floating at draughts 7.800m forward, 8.560m aft in dock water of relative
density 1.015.
The vessel is to load cargo so as to finish on an even keel at a load draught of 8.560m in
dock water.
Two spaces are available: No.1 hold and No.5 hold.
Using the Stability Data Booklet, calculate EACH of the following:
a) The total weight of cargo to load;
b) The weight of cargo to load in each hold.
a) Calculate AMD and Trim
AMD = 8.180m

Trim = 0.760m by stern

Calculate LCF and TMD


LCF

= 66.25 (0.08 x 0.08)


0.10
TMD = 8.560 (0.760 x 66.186)
137.5

= 66.186m foap
= 8.194m

With TMD = 8.194, enter Hydrostatic Tables,


Volume

FW

DW

MCTC FW

MCTC DW
LCB
LCF

= 16742 + (233 x 0.094)


0.10
= 16961 x 1.015

= 16961 m3

= 196.1 + (1.4 x 0.094)


0.10
= 197.4 x 1.015

= 197.4 tm

= 69.54 (0.05 x 0.094)


0.10
= 66.25 (0.08 x 0.094)
0.10

= 17215 t

= 200.4 tm
= 69.49m foap
= 66.17m foap

Calculate LCG
0.760 x 100

LCG

= 17215 x (69.49 LCG)


200.4
= 68.61m foap

Calculate New Displacement with New TMD


TMD
Volume

FW

New DW

= 8.560m (as vessel will complete loading on an even keel draught)


= 17677 + (234 x 0.06)
= 17817 m3
0.10
= 17817 x 1.015
= 18084 t

Calculate total weight of cargo to load


Cargo to load

= 18084 17215

= 869 t

b) Assume w tons of cargo is loaded in No.1 Hold


Amount of cargo loaded in No.5 Hold
= (869 w) tons

ARNAB SARKAR

SQA Stability March 2012


With TMD = 8.560m, calculate the final LCB
LCB

= 69.36 (0.05 x 0.06)


= 69.33m
0.10
As vessel is in even keel, LCB = LCG = 69.33m
Calculate Moments about the LCG
Weights (t)
17215
w
(869 w)
18084

Initial
No.1 Hold
No.5 Hold
Final

(1196120.09 + 97.22w)
18084
or,
w

LCG (m)
68.61
114.48
17.26

Long. Moments (tm)


1181121.15
114.18w
(14998.94 17.26w)
(1196120.09 + 97.22w)

= 69.33
= 593 t

Cargo to load in No.1 Hold 593 t

&

Cargo to load in No.5 Hold

276 t

3. A box shaped vessel of length 110.00m, breadth 16.40m, depth 9.10m is floating in salt
water to an even keel draught of 5.90m.
a) Calculate the righting moment when the vessel is heeled to the angle of deck edge
immersion if the KG is 6.60m.
b) Calculate the angle of loll if the KG is 6.80m.
a) Determine displacement of box shaped vessel
Displacement = 110 x 16.4 x 5.90 x 1.025 = 10910 t
Calculate Angle of Deck Edge Immersion
Freeboard of the box shaped vessel = 9.10 5.90 = 3.20m
Beam
= 8.20m
Tan DEI
= 3.20
8.20
or, DEI
= 21.3
Calculate the KM for the box shaped vessel

KB

= Draught

= 2.95m

BM

110 x 16.43
12 x 110 x 16.4 x 5.90
= KB + BM

= 3.80m

KM

= 2.95 + 3.80 = 6.75m

Calculate GZ using the wall sided formula

GZ
GZ

= {(6.75 6.60) + 0.5 x Tan2 21.3} x Sin 21.3


= 0.16m

Calculate the Righting Moment


Righting Moment

= 0.16 x 10910

b) KG
= 6.80m
GM = 6.75 6.80 = 0.05m
Tan

Loll

= (2 x 0.05)
3.80

= 1745.6 tm

or,

Loll

= 9.2

ARNAB SARKAR

SQA Stability July 2012


1. A vessel carrying timber on deck departs from port with a GM of 0.05m. The stability
of the vessel deteriorates on passage and as a result the vessel settles to an angle of loll
12 to port. Even keel draught 5.700m in salt water.
Investigate the effect of ballasting No.2 DB tanks, filling the tanks one at a time, in the
following order: (1) centre, (2) port, (3) starboard.
Using the Stability Data Booklet calculate EACH of the following:
a)
b)
c)
d)

The initial negative GM prior to ballasting;


The angle of loll on commencing to ballast the centre tank (assume weight negligible);
The GM when the centre tank is full;
The angle of heel when the port tank, TCG 5.00m, is full.

a) Using E.K. of 5.700m, from hydrostatic particulars,


Displacement
KM
KB

BM

= 11625 t
= 8.63m
= 2.95m
= 8.63 2.95 = 5.68m

Determine KG and GM
Tan 12

GM
KG

= (2 x GM)
5.68
= 0.128m
= 8.63 ( 0.128) = 8.758m

b) Calculate Moments about the Keel to determine the KG prior commencing to ballast

Weights (t)
11625
FSM SW
11625
KG
KM
GM

Tan
or,

Loll

Loll

KG (m)
8.758
(1021 x 1.025) =

= 102858.275
11625
= 8.630m
= 8.630 8.848

Vertical Moments (tm)


101811.75
1046.525
102858.275

= 8.848m

= 0.218m

= (2 x 0.218)
5.68
= 15.5

c) Calculate Moments about the Keel to determine the KG when the centre tank is full

Weights (t)
11625
277.8
11902.8
KG

KG (m)
8.758
0.59

= 101975.652
11902.8

= 8.567m

Determine KM for = 11902.8 t


KM

GM

= 8.59 (0.04 x 55)


225
= 8.580 8.567

= 8.580m
= 0.013m

Vertical Moments (tm)


101811.75
163.902
101975.652

ARNAB SARKAR

SQA Stability July 2012


d) Determine the Angle of Heel when the Port tank is full
Calculate Moments about the Keel to determine the KG

Weights (t)
11903
228.6
12131.6
KG

KG (m)
8.567
0.60

= 102110.161
12131.6

Vertical Moments (tm)


101973.001
137.16
102110.161

= 8.417m

Determine KM for = 12131.6 t


KM

GM

= 8.55 (0.03 x 58.6) = 8.542m


224
= 8.542 8.417
= 0.125m

Calculate Moments about the C L to determine the KG

Weights (t)
11903
228.6
12131.6

Dist. from C L (m)


5.00

GG H

= 0.094m

Tan
or,

List

List

= 1143
12131.6

Vertical Moments (tm)


1143
1143

= 0.094
0.125
= 36.9

2. A vessels present particulars are as follows:


Forward draught 8.240m, aft draught 9.240m at an upriver berth in fresh water.
The vessel is to proceed down river to cross a sand bar at the river entrance where the
relative density of the water is 1.018.
During the river passage the following items of deadweight will be consumed:
58 t
19 t
19 t
17 t

of Heavy Fuel Oil


of Diesel Oil
of Diesel Oil
of Fresh Water

from No.3 DB Centre tank


from No.4 DB Port tank
from No.4 DB Stbd. tank
from After Fresh Water tank

Using the Stability Data Booklet, calculate the clearance of the vessel over the bar if
the depth of water over the sand bar is 9.520m.
2. Calculate AMD and Trim
AMD = 8.740m

Trim

Calculate LCF and TMD


LCF

= 65.81 (0.07 x 0.04)


0.10
TMD = 9.240 (1.000 x 65.78)
137.5

= 65.78m foap
= 8.762m

= 1.000m by stern

ARNAB SARKAR

SQA Stability July 2012


Calculate Initial , MCTC, LCB and LCF, using TMD = 8.762m
Initial FW
MCTC FW
LCB
LCF

= 18147 + (236 x 0.062)


0.10
= 202.9 + (2.20 x 0.062)
0.10
= 69.27 (0.05 x 0.062)
0.10
= 65.81 (0.07 x 0.062)
0.10

= 18293 t
= 204.3 tm
= 69.24m foap
= 65.77m foap

Calculate Initial LCG


1.000 x 100

LCG

= 18293 x (69.24 LCG)


204.3
= 68.12m foap

Calculate Longitudinal Moments about the LCG


Initial
() HFO
() DO
() DO
() FW
Final

Weights (t)
18293
58
19
19
17
18180

LCG (m)
68.12
57.02
35.66
35.50
28.67

Long. Moments (tm)


1246119.16
3307.16
677.54
674.50
487.39
1240972.57

Determine 1.018

Final
1.018

= 18180 t
= 18180
1.018

= 17859 t

Calculate TMD, LCB, LCF and MCTC from hydrostatic particulars, using = 17859 t
TMD

= 8.50 + (0.10 x 182)


234
= 69.36 (0.05 x 182)
234
= 65.95 (0.08 x 182)
234
= 201.4 + (1.20 x 182)
234

LCB
LCF
MCTC

= 8.578m
= 69.32m foap
= 65.89m foap
= 202.3 tm

Determine the Final Trim


Trim

= 17859 x (69.32 68.26)


202.3

COT A = 0.936 x 65.89


137.5
COT F = 0.936 0.449

= 93.58cms or 0.936m by stern


as (LCB> LCG)
= 0.449m
= 0.487m

Calculate the Final Draughts


TMD
COT F/A

()

Forward
8.578m
0.487m
8.091m

(+)

Aft
8.578m
0.449m
9.027m

ARNAB SARKAR

SQA Stability July 2012


Determine clearance above the sand bar
Clearance

= 9.520 9.027

= 0.493m

3. A vessel is floating in dock water of R.D. 1.017.


Present draughts:
Forward 8.060m, Midship (Port) 8.620m, Midship (Starboard) 8.580m, Aft 9.260m
The draught marks are displaced as follows:
Forward
Aft
Midship

: 1.32m forward of the FP


: 2.08m forward of the AP
: 0.66m of the amidship line

The Stability Data Booklet provides the necessary hydrostatic data for the vessel.
By completion of the Worksheet Q3 and showing any additional calculations in the
answer book, determine the vessels displacement.
3. Draught forward
Trim
FP Correction
Draught at FP
Draught aft
AP Correction
Draught at AP
True Trim

= 8.060m
= 9.260 8.060
= 1.32 x 1.200
137.5
= 8.060 0.012
= 9.260m
= 2.08 x 1.200
137.5
= 9.260 + 0.018
= 9.278 8.048

Draught (M) Port


Draught (M) Stbd

= 8.620m
= 8.580m

Draught (M) Mean

= 8.600m

= 1.200m by stern
= 0.012m
= 8.048m
= 0.018m
= 9.278m
= 1.230m by stern

Amidships Line Corr. = 0.66 x 1.230


= 0.006m
137.5
Draught at Midships = 8.600 0.006
= 8.594m
Corr. (M) draught = 8.048 + (6 x 8.594) + 9.278
8

= 8.611m

From hydrostatic particulars using draught 8.611m


TPC SW

= 24.13 + (0.05 x 0.011)


0.10
LCF
= 65.87 (0.06 x 0.011)
0.10
Displacement SW
= 18359 + (242 x 0.011)
0.10
Distance of LCF from Midships
= 68.75 65.86

1st Trim Correction

MCTC 9.111m

= 24.14tm
= 65.86m foap
= 18386 t
= 2.89m

= 2.89 x 123.0 x 24.14


137.5

= 213.0 + (2.2 x 0.011)


0.10

= 62.41 t

= 213.2 tm

ARNAB SARKAR

SQA Stability July 2012


MCTC 8.110m

= 201.0 + (1.4 x 0.011)


0.10
2nd Trim Correction = 50 x (1.230)2 x 12.0
137.5

= 201.2 tm

= 6.600 t

Corrected
Dock Water

= 18386 + 62.41 + 6.60


= 18455.01 x 1.017
1.025

= 18455.01 t
= 18311 t

ARNAB SARKAR

SQA Stability December 2012


1. A vessel, length 137.5m arrives off an upriver berth in fresh water with the following
particulars:
Forward draught 3.826m

Aft draught 4.248m

GM 0.15m

During berthing operations, a starboard side tank is damaged resulting in a loss of 120
t of water ballast, KG 9.12m, LCG 57.87m, TCG 5.10m and the creation of a
(corrected) FSM of 298 m4.
Using the Hydrostatic Particulars contained in the Stability Data Booklet, calculate
EACH of the following:
a) The resultant draughts, forward and aft;
b) The resultant angle and direction of heel.
a) Calculate AMD and Trim
AMD = 4.037m

Trim

= 0.422m by stern

Calculate LCF and TMD


LCF

= 69.81 (0.07 x 0.037)


0.10
TMD = 4.248 (0.422 x 69.78)
137.5

= 69.78m foap
= 4.034m

Calculate Initial , MCTC, LCB and KM, using TMD = 4.034m


Initial
MCTC
LCB
KM

KG

= 7713 + (210 x 0.034)


0.10
= 146.0 + (0.90 x 0.034)
0.10
= 71.15 (0.03 x 0.034)
0.10
= 9.96 (0.13 x 0.034)
0.10
= 9.92 0.15

= 7784.4 t
= 146.3 tm
= 71.14m foap
= 9.92m
= 9.77m

Calculate Initial LCG


0.422 x 100

LCG

= 7784 x (71.14 LCG)


146.3
= 70.35m foap

Calculate Longitudinal Moments about the LCG


Initial
() Ballast
Final

Weights (t)
7784
120
7664

LCG (m)
70.35
57.87

LCG

TMD

= 3.90 + (0.10 x 159)


208
= 145.1 + (0.90 x 159)
208

540660
7664

Long. Moments (tm)


547604.4
6944.4
540660

= 70.55m foap

Calculate TMD, MCTC, LCB, LCF and KM from hydrostatic particulars, using = 7664 t

MCTC

= 3.976m
= 145.8 tm

ARNAB SARKAR

SQA Stability December 2012


LCB

= 71.18 (0.03 x 159)


208
= 69.88 (0.07 x 159)
208
= 10.11 (0.15 x 159)
208

LCF
KM

= 71.16m foap
= 69.83m foap
= 10.00m

Calculate Trim
Trim

= 7664 x (71.16 70.55)


145.8

COT A = 0.321 x 69.83


137.5
COT F = 0.321 0.163

= 32.06cms or 0.321m by stern


as (LCB> LCG)
= 0.163m
= 0.158m

Calculate the Final Draughts


TMD
COT F/A

()

Forward
3.976m
0.158m
3.818m

(+)

Aft
3.976m
0.163m
4.139m

b) Calculate Moments about the Keel and C L


Weights (t)

KG (m)

Vertical Mom. (tm)

7784
() 120
(+) FSM
7664

9.77
9.12

76049.68
() 1094.4
(+) 298.0
75253.28

Final KG

= 75253.28
7664
= 612
7664

GG H

Dist. from C L
P
S
5.10

Trans. Mom.(tm)
P
S
612
612

= 9.82m
= 0.080m

Calculate Final GM
GM

= 10.00 9.82 = 0.18m

Calculate the resultant heel


Tan

Heel

Heel

= 0.080
0.18
= 24 to Port

2. A vessel is to enter drydock in dock water of relative density 1.007.


Drafts:

Forward 3.220m

Aft 4.920m

KG 9.52m

a) Using the Stability Data Booklet, calculate the maximum trim at which the vessel can
enter drydock so as to maintain a GM of at least 0.10m at the critical instant.
b) Calculate the weight of ballast to transfer from the After Peak to the Fore Peak in
order to bring the vessel to the maximum trim calculated in Q2(a).
a) Calculate AMD and Trim
AMD = 4.070m

Trim

= 1.700m by stern

ARNAB SARKAR

SQA Stability December 2012


Calculate LCF and TMD
LCF

= 69.81 (0.07 x 0.07)


0.10
TMD = 4.92 (1.700 x 69.76)
137.5

= 69.76m foap
= 4.058m

Calculate Initial , MCTC, LCB, LCF and KM, from hydrostatic particulars,
Initial

FW

MCTC FW
LCF
LCB
KM

= 7713 + (210 x 0.058)


0.10
= 146.0 + (0.90 x 0.058)
0.10
= 69.81 (0.07 x 0.058)
0.10
= 71.15 (0.03 x 0.058)
0.10
= 9.96 (0.03 x 0.058)
0.10

= 7835 t
= 146.5 tm
= 69.77m foap
= 71.13m foap
= 9.88m

Calculate the Loss of GM


KM
KG F
GM F
Required GM
Loss of GM

= 9.88m
= 9.52m
= 0.36m
= 0.10m
= 0.26m

Calculate the P Force


0.26

= P x 9.88
7835 x 1.007
= 207.6 t

Calculate the maximum trim at which the vessel can enter the drydock
207.6 = Trim x 146.5 x 1.007
69.77
Max. Trim = 98.2cms

b) Assume that w tons of water ballast needs to be transferred from the AP tank to the FP
tank.
Determine the Initial LCG
1.700 x 100
or,

LCG

= 7835 x 1.007 (71.13 LCG)


146.5 x 1.007
= 67.95m foap

Calculate Moments about the LCG


Initial
(+) Ballast
() Ballast
Final
LCG

Weights (t)
(7835 x 1.007)
+w
w
7890

= (536125.5 + 127.49 w)
7890

LCG (m)
67.95
130.56
3.07

Long. Moments (tm)


536125.5
130.56 w
3.07 w
(536125.5 + 127.49 w)

ARNAB SARKAR

SQA Stability December 2012

98.2

7890

or,

83.2 t

(71.13 536125.5 +127.49 w)


7890
146.5 x 1.007

Amount of water ballast to be transferred from AP to FP tank = 83.2 t

3. A box shaped vessel floating upright on an even keel in salt water has the following
particulars:
Length 162.00m

Breadth 30.00m

Draught 8.800m

KG 10.850m

The vessel has two longitudinal bulkheads each 6.00m from the side of the vessel.
Calculate the angle of heel if a midship side compartment 27.00m is bilged.
3. Calculate Sinkage and Bilged Draught
Volume of Buoyancy lost
or, (27 x 6 x 8.800)
or, Sinkage

=
=
=

Bilged draught

Volume of Buoyancy gained


{(162 x 30) (27 x 6)} x Sinkage
1425.6
= 0.303m
4698
8.800 + 0.303 = 9.103m

Calculate new location of LCF from the side XX


Calculate moments of area about the axis XX
Total Area
Bilged Area

Area (m2)
162 x 30 = 4860
27 x 6 = 162
4698

New location of LCF


Distance BB H

Distance from axis XX (m)


15.0
3.0
()

= 15.414m
= 15.414 15.0 = 0.414m

Calculate Moment of Inertia (I) about the new LCF


= I XX Ad2
3
3
2
LL = [(162 x 30 ) (27 x 6 )] [{(162 x 30) (27 x 6)} x 15.414 ]
3
3
or, I LL = (1458000 1944) 1116204.378
or, I LL = 339851.622 m4
I LL
or, I

Calculate bilged BM, KB, KM and GM


BM

339851.622
(162 x 30 x 8.800)
KB
= 9.103
2
KM = 7.946 + 4.552
GM = 12.498 10.850
Calculate List

Tan = 0.414
1.648
List = 14.1

= 7.946m
= 4.552m
= 12.498m
= 1.648m

Moments (m4)
72900
486
72414

ARNAB SARKAR

SQA Stability March 2013


1. A box shaped vessel floating on even keel in dock water of R.D. 1.011 has the following
particulars:
Length 120.00m
Draught 7.800m

Breadth 22.00m
MCTC (salt water) 290

There is an empty watertight forward end compartment, length 12.00m, height 6.50m,
extending the full width of the vessel.
Calculate the draughts forward and aft, if this compartment is bilged.
1. Calculate the Bilged TMD
Volume of the vessel before bilging
= 120 x 22 x 7.800
Displacement of the vessel before bilging = 20592 x 1.011

= 20592 m3
= 20818.5 t

Permeability of the bilged compartment

= 1.00

Volume of the forward end compartment


Intact water plane area

= 12 x 22 x 6.500
= 120 x 22

= 1716 m3
= 2640 m2

Sinkage caused to bilging

= 1716
= 0.650m
2640
= 7.800 + 0.650
= 8.450m

Bilged TMD

Calculate the Trimming Moment


Calculating moments of volume about the AP after bilging,
Volume (m3)
120 x 22 x 8.450
() 1716
20592

Distance from AP (m)


60.0
114.0

Moments (m4)
1338480.0
() 195624
1142856.0

B H = 55.500m
BB H = 60 55.500 = 4.500m
Trimming Moment = 20818.5 x 4.500 = 93683.25 tm by forward
Calculate COT after bilging
COT

= 93683.25 x 1.025 = 327.5cm or 3.275m


290 x 1.011
COT A
= 3.275 x 60
= 1.638m
120
COT F
= 3.275 1.638
= 1.637m
Calculate the Final Draughts of the vessel
Forward Draught
Aft Draught

= 8.450 + 1.637 = 10.087m


= 8.450 1.637 = 6.813m

2. A vessel completes under deck loading in salt water with the following particulars:
Displacement 14115 tonne

KG 8.00m

The Stability Data Booklet provides the necessary data for the vessel.
During the passage 190 tonne of Heavy Fuel Oil (R.D. 0.81) is consumed from No.3
D.B. tank centre which was full on departure (use KG of tank for consumption
purposes).

ARNAB SARKAR

SQA Stability March 2013


Calculate the maximum weight of timber to load on deck KG 13.20m assuming 15%
water absorption during the passage in order to arrive at the destination with the
minimum GM (0.05m) allowed under the Load Line Regulations.
Note: Assume KM constant.
2. Determine the KM of the vessel from hydrostatic particulars, using = 14115 t
KM

= 8.36m

Assume that w tonnes of timber can be loaded to meet the required condition.
Considering water absorption by the timber deck cargo, extra weight added = 0.15 w tons
Calculate Moments about the Keel to determine the Final KG
Initial
Cargo
Water Absorption
() HFO
FSM HFO
Final

Weights (t)
14115
w
0.15 w
() 190

KG (m)
8.00
13.2
13.2
0.60
1142 x 0.81

(13925 +1.15 w)

Final KG

(113731.02 + 15.18 w) m
(13925 +1.15 w)

8.36 0.05

(113731.02 + 15.18 w)
(13925 +1.15 w)

or, 115716.75 +9.56w =

113731.02 + 15.18w

or,

353.3 t

Weight of timber to load

Vertical Moments (tm)


112920
13.2 w
1.98 w
() 114
(+) 925.02
(113731.02 + 15.18 w)

= 353.3 t

3. A vessel is to load a cargo of grain (stowage factor 1.65 m3/t).


Initial displacement 6020 tonne
Initial KG 6.68m
All five holds are to be loaded full of grain.
The tween decks are to be loaded as follows:
No.1 TD
No.2 TD
No.3 TD
No.4 TD

Full
Part Full Ullage 2.50m
Part Full Ullage 1.25m
Full

The Stability Data Booklet provides the necessary cargo compartment data for the
vessel.
a) Using the Maximum Permissible Grain Heeling Moment Table included in the Stability
Data Booklet, determine whether the vessel complies with the minimum criteria
specified in the International Grain Code (IMO).
b) Calculate the ships approximate angle of heel in the event of the grain shifting as
assumed by the International Grain Code (IMO).
a) Initial
SF

= 6020 t
= 1.65 m3/t

Initial KG

= 6.68m

ARNAB SARKAR

SQA Stability March 2013


Calculate the Final , KG & VHM
Comp.

Ullage
(m)

Volume
(m3)

SF
(m3/t)

Initial

Vert. Mom.
(tm)
40214

VHM
(tm)

6020

KG
(m)
6.68

Tonnes

No.1 TD

Full

1695

1.65

1027

11.26

11564

352

No.2 TD

2.50

1062

1.65

644

10.44

6723

3160

No.3 TD

1.25

1476

1.65

895

10.47

9371

1547

No.4 TD

Full

1674

1.65

1015

10.57

10729

604

No.1 Hold

Full

2215

1.65

1342

5.09

6831

410

No.2 Hold

Full

4672

1.65

2832

4.95

14018

1285

No.3 Hold

Full

1742

1.65

1056

4.94

5217

475

No.4 Hold

Full

3474

1.65

2106

4.95

10425

910

No.5 Hold

Full

2605

1.65

1579

8.76

13832

455

18516

6.96

128924

9198

Final
Calculate AHM
AHM = 9198
1.65

= 5575tm

Calculate Maximum Permissible Heeling Moment


= 18516 t

KG F = 6.96m

Interpolation

KG F = 6.90

KG F = 6.96

KG F = 7.00

19000

7157

6905.0

6737

18516
18500

6660.7
6898

6652.6

6489

Holding = 19000 t constant, using variable KG F = 6.96m,


7157 (420 x 0.06) = 6905.0 tm
0.10
Holding = 18500 t constant, using variable KG F = 6.96m,
6898 (409 x 0.06) = 6652.6 tm
0.10
Holding KG F = 6.96m constant, using variable = 18516 t,
6652.6 + (252.4 x 16) = 6660.7 tm
500
PHM for = 18516 t & KG F = 6.96m = 6660.7 tm
Vessel complies with the minimum criteria if she is upright during sailing.

(b) Calculate approximate angle of list in event of grain shift


Angle of Heel = 5575 x 12 = 10.0
6660.7

You might also like